If visible light has more energy than microwaves, why isn't visible light dangerous?












30














Light waves are a type of electromagnetic wave and they fall between 600-700 nm long. Microwaves are less energetic but seem to be more dangerous than visible light. Is visible light dangerous at all and why not?










share|cite|improve this question




















  • 43




    how are microwaves dangerous?
    – ZeroTheHero
    yesterday






  • 4




    @ZeroTheHero : depends on the intensity of microwave radiation. If it can cook our meals, it can cook us :-(. For example, it can cause blindness and sterility through thermal effect. See osha.gov/SLTC/radiofrequencyradiation/hazards.html
    – akhmeteli
    yesterday






  • 12




    @akhmeteli The same amount of energy delivered through visible light will do comparable or even more significant damage (the only thing that would make visible light less dangerous would be part of the energy being reflected). As will falling into a pot of boiling water. That doesn't make microwaves "more dangerous than visible light". Of couse there are thermal effects, but those don't care about the wavelength besides determining how much of energy is absorbed.
    – Luaan
    yesterday






  • 16




    I'm pretty sure that if you stick your hand in a box with a 900W visible light generator, you get at least as serious damage as if you put your hand in a 900W microwave oven
    – Suppen
    yesterday






  • 7




    This is an odd question. Let me pose a question to you: A lead bullet is much more dense than a grand piano, and yet if you drop a grand piano on me from ten meters up, it will be much more harmful than if you drop a lead bullet on me from ten meters up. So are pianos more dangerous than bullets despite their low density? ("Should we have better piano control?" is a political question of course.)
    – Eric Lippert
    20 hours ago


















30














Light waves are a type of electromagnetic wave and they fall between 600-700 nm long. Microwaves are less energetic but seem to be more dangerous than visible light. Is visible light dangerous at all and why not?










share|cite|improve this question




















  • 43




    how are microwaves dangerous?
    – ZeroTheHero
    yesterday






  • 4




    @ZeroTheHero : depends on the intensity of microwave radiation. If it can cook our meals, it can cook us :-(. For example, it can cause blindness and sterility through thermal effect. See osha.gov/SLTC/radiofrequencyradiation/hazards.html
    – akhmeteli
    yesterday






  • 12




    @akhmeteli The same amount of energy delivered through visible light will do comparable or even more significant damage (the only thing that would make visible light less dangerous would be part of the energy being reflected). As will falling into a pot of boiling water. That doesn't make microwaves "more dangerous than visible light". Of couse there are thermal effects, but those don't care about the wavelength besides determining how much of energy is absorbed.
    – Luaan
    yesterday






  • 16




    I'm pretty sure that if you stick your hand in a box with a 900W visible light generator, you get at least as serious damage as if you put your hand in a 900W microwave oven
    – Suppen
    yesterday






  • 7




    This is an odd question. Let me pose a question to you: A lead bullet is much more dense than a grand piano, and yet if you drop a grand piano on me from ten meters up, it will be much more harmful than if you drop a lead bullet on me from ten meters up. So are pianos more dangerous than bullets despite their low density? ("Should we have better piano control?" is a political question of course.)
    – Eric Lippert
    20 hours ago
















30












30








30


4





Light waves are a type of electromagnetic wave and they fall between 600-700 nm long. Microwaves are less energetic but seem to be more dangerous than visible light. Is visible light dangerous at all and why not?










share|cite|improve this question















Light waves are a type of electromagnetic wave and they fall between 600-700 nm long. Microwaves are less energetic but seem to be more dangerous than visible light. Is visible light dangerous at all and why not?







energy visible-light electromagnetic-radiation estimation microwaves






share|cite|improve this question















share|cite|improve this question













share|cite|improve this question




share|cite|improve this question








edited yesterday









Qmechanic

102k121831159




102k121831159










asked yesterday









susesuse

29737




29737








  • 43




    how are microwaves dangerous?
    – ZeroTheHero
    yesterday






  • 4




    @ZeroTheHero : depends on the intensity of microwave radiation. If it can cook our meals, it can cook us :-(. For example, it can cause blindness and sterility through thermal effect. See osha.gov/SLTC/radiofrequencyradiation/hazards.html
    – akhmeteli
    yesterday






  • 12




    @akhmeteli The same amount of energy delivered through visible light will do comparable or even more significant damage (the only thing that would make visible light less dangerous would be part of the energy being reflected). As will falling into a pot of boiling water. That doesn't make microwaves "more dangerous than visible light". Of couse there are thermal effects, but those don't care about the wavelength besides determining how much of energy is absorbed.
    – Luaan
    yesterday






  • 16




    I'm pretty sure that if you stick your hand in a box with a 900W visible light generator, you get at least as serious damage as if you put your hand in a 900W microwave oven
    – Suppen
    yesterday






  • 7




    This is an odd question. Let me pose a question to you: A lead bullet is much more dense than a grand piano, and yet if you drop a grand piano on me from ten meters up, it will be much more harmful than if you drop a lead bullet on me from ten meters up. So are pianos more dangerous than bullets despite their low density? ("Should we have better piano control?" is a political question of course.)
    – Eric Lippert
    20 hours ago
















  • 43




    how are microwaves dangerous?
    – ZeroTheHero
    yesterday






  • 4




    @ZeroTheHero : depends on the intensity of microwave radiation. If it can cook our meals, it can cook us :-(. For example, it can cause blindness and sterility through thermal effect. See osha.gov/SLTC/radiofrequencyradiation/hazards.html
    – akhmeteli
    yesterday






  • 12




    @akhmeteli The same amount of energy delivered through visible light will do comparable or even more significant damage (the only thing that would make visible light less dangerous would be part of the energy being reflected). As will falling into a pot of boiling water. That doesn't make microwaves "more dangerous than visible light". Of couse there are thermal effects, but those don't care about the wavelength besides determining how much of energy is absorbed.
    – Luaan
    yesterday






  • 16




    I'm pretty sure that if you stick your hand in a box with a 900W visible light generator, you get at least as serious damage as if you put your hand in a 900W microwave oven
    – Suppen
    yesterday






  • 7




    This is an odd question. Let me pose a question to you: A lead bullet is much more dense than a grand piano, and yet if you drop a grand piano on me from ten meters up, it will be much more harmful than if you drop a lead bullet on me from ten meters up. So are pianos more dangerous than bullets despite their low density? ("Should we have better piano control?" is a political question of course.)
    – Eric Lippert
    20 hours ago










43




43




how are microwaves dangerous?
– ZeroTheHero
yesterday




how are microwaves dangerous?
– ZeroTheHero
yesterday




4




4




@ZeroTheHero : depends on the intensity of microwave radiation. If it can cook our meals, it can cook us :-(. For example, it can cause blindness and sterility through thermal effect. See osha.gov/SLTC/radiofrequencyradiation/hazards.html
– akhmeteli
yesterday




@ZeroTheHero : depends on the intensity of microwave radiation. If it can cook our meals, it can cook us :-(. For example, it can cause blindness and sterility through thermal effect. See osha.gov/SLTC/radiofrequencyradiation/hazards.html
– akhmeteli
yesterday




12




12




@akhmeteli The same amount of energy delivered through visible light will do comparable or even more significant damage (the only thing that would make visible light less dangerous would be part of the energy being reflected). As will falling into a pot of boiling water. That doesn't make microwaves "more dangerous than visible light". Of couse there are thermal effects, but those don't care about the wavelength besides determining how much of energy is absorbed.
– Luaan
yesterday




@akhmeteli The same amount of energy delivered through visible light will do comparable or even more significant damage (the only thing that would make visible light less dangerous would be part of the energy being reflected). As will falling into a pot of boiling water. That doesn't make microwaves "more dangerous than visible light". Of couse there are thermal effects, but those don't care about the wavelength besides determining how much of energy is absorbed.
– Luaan
yesterday




16




16




I'm pretty sure that if you stick your hand in a box with a 900W visible light generator, you get at least as serious damage as if you put your hand in a 900W microwave oven
– Suppen
yesterday




I'm pretty sure that if you stick your hand in a box with a 900W visible light generator, you get at least as serious damage as if you put your hand in a 900W microwave oven
– Suppen
yesterday




7




7




This is an odd question. Let me pose a question to you: A lead bullet is much more dense than a grand piano, and yet if you drop a grand piano on me from ten meters up, it will be much more harmful than if you drop a lead bullet on me from ten meters up. So are pianos more dangerous than bullets despite their low density? ("Should we have better piano control?" is a political question of course.)
– Eric Lippert
20 hours ago






This is an odd question. Let me pose a question to you: A lead bullet is much more dense than a grand piano, and yet if you drop a grand piano on me from ten meters up, it will be much more harmful than if you drop a lead bullet on me from ten meters up. So are pianos more dangerous than bullets despite their low density? ("Should we have better piano control?" is a political question of course.)
– Eric Lippert
20 hours ago












10 Answers
10






active

oldest

votes


















55














Your question contains a premise that is false: Microwaves do not have less energy than visible light per se. They only have less energy per photon, as per the Planck--Einstein relation, $E = hf$. In other words, you can raise the power of electromagnetic radiation to a dangerous level at any wavelength, if only you generate enough photons -- as your microwave oven does.



That very much includes visible light. You can easily verify this by waiting for a sunny day, getting out your magnifying glass, and using it to focus sunlight onto your forearm. (Just be sure to stop when you start noticing the smell of burning hair.) In conclusion, then, sunlight is dangerous!



EDIT: As user JMac suggests in his comment below, better focus the sunlight on a piece of paper first. Watch it char and maybe even burn, and then think twice before trying it on your arm. Also on the subject of offering words of caution, make sure there's nothing around that piece of paper that can burn.






share|cite|improve this answer










New contributor




Thomas Blankenhorn is a new contributor to this site. Take care in asking for clarification, commenting, and answering.
Check out our Code of Conduct.














  • 12




    "You can easily verify this by waiting for a sunny day, getting out your magnifying glass, and using it to focus sunlight onto your forearm." Nope. Don't do this. It's not good for you, unless you like burns, sunburn and melanoma.
    – wizzwizz4
    yesterday






  • 7




    I was half-joking, and wizzwizz4 makes a fair point. While the experiment does demonstrate that visible light can be harmful, the flip side is, don't do it if you don't like pain! No to the specific point about sunburn and melanoma, though. These are both caused by ionizing radiation (ultraviolet and shorter wavelengths),. Most of that gets filtered out by the glass (or plastic) that lenses are made of. During the few seconds until one feels the burn, twitches, and thereby ends the experiment, one won't accumulate enough to cause lasting harm.
    – Thomas Blankenhorn
    yesterday








  • 2




    Didn't know that plastic lenses also filtered out UV. (I assume it's only some plastics.) Thanks for the information!
    – wizzwizz4
    yesterday






  • 20




    Safety comment - For any readers who may not realize this - The note about burning hair is not a joke. If you focus sunlight on your skin with a magnifying glass, you will burn yourself severely. DO NOT DO THIS.
    – TypeIA
    23 hours ago






  • 6




    You may want to suggest that they try the experiment on something like a piece of paper first. You can quickly figure out a good reason on your own for why you wouldn't want to focus it on your arm.
    – JMac
    21 hours ago



















41














If you stare at the Sun you’ll go blind. And if you spend a lot of time in the sun, you’re likely to get skin cancers. So visible light seems plenty dangerous to me.



Some of the damage may actually be from infrared and ultraviolet light, but these are close in frequency to visible light and very far from microwaves.



By the way, the intensity also matters, not just the frequency. In terms of photons, it matters not only how energetic each photon is, but also how many photons are arriving per second.






share|cite|improve this answer

















  • 16




    not to forget laser guns,
    – anna v
    yesterday






  • 22




    This answer could be improved by addressing the OP's comparison with microwaves and discussing ways in which microwaves are dangerous (heating) and are not (cancer).
    – Ben Crowell
    yesterday






  • 4




    Could also compare the wattage of a microwave vs sunlight
    – JollyJoker
    yesterday








  • 6




    @BenCrowell The thing is, there's nothing really dangerous about microwaves - the only damage they do is through heating, which doesn't depend on the wavelength as long as the material absorbs most of the energy anyway. The same amount of energy from visible light does far more damage. But adding it to the answer is probably a good idea, since it seems suse doesn't understand that part.
    – Luaan
    yesterday






  • 10




    "And if you spend a lot of time in the sun, you’re likely to get skin cancers. So visible light seems plenty dangerous to me." - Skin cancer is caused by the UV radiation the sun emits, not visible light.
    – marcelm
    yesterday



















32














The dose (or, in this case, the intensity) makes the poison. You're constantly exposed to microwaves since that's what 99% of wireless communication devices use, and you're also constantly exposed to visible light unless you sleep in an isolation tank. Both can be dangerous if you increase the intensity sufficiently.






share|cite|improve this answer

















  • 1




    For that matter, you are constantly exposed to microwaves from the sun and, for that matter, the CMB. Just at very low levels.
    – WhatRoughBeast
    23 hours ago






  • 1




    Let me put some numbers behind that. A standard microwave oven puts 650W of electromagnetic energy into whatever you put inside it (many ovens actually do more these days). A wifi router is limited to 4 watts EIRP of RF radiation, which falls off with distance on the familiar inverse-square law. A modern bicycle headlamp, operating from a 3-watt dynamo, uses LEDs with a thermal efficiency of about 50%, so emits about 1.5 watts of visible light energy - and that's pretty bright. Now imagine looking at 650W of visible light…
    – Chromatix
    2 hours ago



















16














Assuming the intensity is the same, microwaves are more dangerous than visible light because they penetrate the skin to a greater depth (1-2 cm; more info is in Wikipedia).



Humans have more adaptation to visible light than to microwave radiation, because they were exposed to light for millions of years. This is expressed in two ways:




  • Proteins in the epidermis (outer skin layer) are more resistant to heat than those in the deeper skin layers

  • There are more nerve endings in the outer layers of skin, so dangerous heating by visible light causes more pain, urging you to escape the dangerous situation


Oh, and the most obvious difference: visible light is visible. Dangerous levels of visible light (e.g. in a solar cooker), to our eyes, look blinding and obviously dangerous. Dangerous levels of microwave radiation are invisible.






share|cite|improve this answer























  • In general, anything that penetrates more is less dangerous; since it means it's less ionising. For example, gamma radiation; when it does ionise does a whole lot of damage... however it's such high energy, most of them are able to pass through the body without impacting it at all. While something like an alpha particle (sure, it's not EM, but concept applies) becomes VERY dangerous because it doesn't penetrate at all. Obviously, reflecting and passing through are the same in that the energy isn't absorbed by the organs; meaning no ionisation occurs.
    – UKMonkey
    yesterday








  • 1




    Wikipedia doesn't seem to support your claim. For instance, the microwave intensity found to cause cataracts in rabbits is 150 mW/cm2 for 100 minutes. That is roughly equal to the intensity of natural sunlight. I believe staring at the Sun will make you blind much faster.
    – Dmitry Grigoryev
    yesterday






  • 2




    @UKMonkey Re "alpha is more dangerous than gamma because it penetrates less": Simply not true: "exposure to most alpha particles originating outside the body is not a serious hazard." Or maybe a Briton would trust the BBC. The same applies to light: The skin absorbs/reflects lots of it and protects underlying tissue; while microwaves can penetrate it and reach living cells.
    – Peter A. Schneider
    yesterday












  • @PeterA.Schneider Sure - emphasis on outside the body. Inside the body is a very different story. That's because our skin has been very well adapted to protect us. The point is that you can't just say "it's more penetrating -> it's more dangerous" EM Radiation isn't a gun - if it goes through you, it's harmless. Now there's a sweet spot where all the radiation is absorbed in the body, and it penetrates some distance - which is how cancer treatment works; however attempting to suggest that masters in physics is related to the media is insulting, and I'd be thankful if you could refrain.
    – UKMonkey
    yesterday










  • @UKMonkey I didn't mean to insult, and I didn't know you had a Master's degree in Physics.-- "Outside the body": Well, that's the setting of the question: Obviously the radiation (light or microwave) is meant to originate outside the body, and the main reason light does less harm is that it doesn't penetrate the skin much.
    – Peter A. Schneider
    yesterday





















10














The danger of electromagnetic waves is a function of the photon energy, the intensity of the source and your distance from it, and the qualitative nature of the interaction of a specific frequency with organic matter.



That latter bit is very complex. The visible spectrum, down to about infra-red, doesn't penetrate the top layer of skin, or most clothing, so for the most part its interaction is limited to heating. Strong infrared can certainly cause burns. Strong visible light can certainly cause eye damage. But very high-intensity sources of visible light are rare in daily life, and very notably, we can see them, and avoid them.



Further down in the energy spectrum (longer wavelength) you get the "millimeter waves" of airport scanners, which can penetrate clothing but not skin, and then you're into the microwaves, uhf and vfh radio waves, and then the radio waves called shortwaves (high frequency), medium waves and longwaves (low frequency). Microwaves can penetrate into flesh, and radio frequencies can entirely traverse a human body, and these can cause very, very severe deep-tissue burns. Certain frequencies can also interfere with cardiac rhythm, which can be as fatal as it sounds.



Your home wifi equipment produces microwaves on pretty much the same frequencies as your microwave oven, but at milliwatt power and dispersed in all directions. The oven dumps hundreds of watts into a small enclosed space. That's the difference.



And we live in a sea of radio waves from microwave (cell phone, wifi) though uhf and vhf (two-way radios, broadcast TV and FM) and lower (broadcast radio). The key is the power. If you grab the antenna of your uncle's 500-W ham radio when he keys the mic, or climb the tower of a multi-kilowatt tv station, you'll get hurt, maybe very badly. But going about your normal business, you're probably absorbing less than a milliwatt of radio energy. And the only effect is heating, so it's little different than being in a room that's very slightly warmer.



Now, moving upward in energy from the visible spectrum, you get ultraviolet, x-rays and then a vast spectrum of increasing energy gamma. Not only can they penetrate into flesh, they have a very specific dirty trick: they have enough photon energy to ionize molecules, and when that happens to our dna and proteins, we start to have very bad days. This is a very specific capability that begins at a certain energy threshold.



Microwaves, as you've remarked, are in the opposite direction from UV, X-rays and gamma: lower photon energy, longer wavelength. They cannot duplicate the ionization danger of higher energies, no matter how intense their sources are.






share|cite|improve this answer








New contributor




CCTO is a new contributor to this site. Take care in asking for clarification, commenting, and answering.
Check out our Code of Conduct.


























    3














    Other answers already point out the matter of intensity. If you have a 1kW microwave that cooks your chicken and so call microwaves dangerous, you can equally cook chicken with 1kW visible light bulb. The difference is mostly how well and how deep the absorption goes, but the amount of energy is the same if the absorbed power is the same.



    What makes the difference:




    • Absorption coefficient; how WELL the light is absorbed. So, a black chicken will cook well in strong visible light, but a white chicken will require higher power, because it reflects more. Microwaves work well because they penetrate deeper before they get absorbed (due to longer wavelength), but also absorb WELL because the frequency is tuned near the resonance for water molecules.

    • Resonances; If the wavelength matches exactly to one of the transition frequencies for molecules/atoms in the target, most of the energy is selectively absorbed just by those molecules. So, if you tuned your light specifically to a transition that breaks some specific bonds, or heat just specific tissues. This can do more damage because it may change chemistry, but luckily, breaking bonds requires quite high frequencies - see next case below. With MW and IR, you will still end up just heating the sample if you find a resonance (resonances in MW, IR and visible are mostly vibrational and rotational transitions, not bond changing, except for red-colored substances reacting to visible light, which you notice when red dyes bleach quickly in intense light).

    • Ionization; If the energy of a SINGLE photon is enough to kick off an electron from a molecule/atom, then it's dangerous because it's actively affecting the chemistry (note that this is similar result than the resonant case above, but instead of having a precise frequency, it has way too much energy, with similar results). This is what is called ionising radiation (gamma/X-rays, down to UV range).


    Note that resonance just means good absorbtion, nothing mystical. Water is mostly transparent for visible light because no significant vibrations of water molecule fall in this range - most are in IR and microwave, and there is another absorption range in UV.



    Rule of thumb: microwaves, IR and visible light just heat you up. It's the heat is sufficient to raise the temperature into danger zone, it's dangerous, otherwise it's harmless. Only intensity matters (Watts per square meter), not the frequency.
    Ionizing radiation (UV/X-ray/gamma) are dangerous because of chemical damage even at low intensity.



    Microwaves are NOT ionizing radiation, so the wireless and mobile signals do absolutely nothing - the power is way too low, otherwise you'd need to charge your phone every 5 minutes.






    share|cite|improve this answer





























      2














      It's more accurate to say microwave ovens are dangerous. Then again, so is visible light.



      It's a question not of photon energy, but total energy. A typical microwave outputs on the order of 1 kilowatt of electromagnetic radiation which is almost entirely absorbed by the food within.



      By comparison, the solar power at earth's surface, at maximum, is around 1 kilowatt per square meter. If it's cloudy, not at the equator, or not noon, it will be less. Most foods have a surface area of much less than a square meter, so the total electromagnetic radiation power received by something sitting in the sun is much less than a microwave oven.



      For a fair comparison, what do you think would happen if a magnifying glass with an area of one square meter, on a very sunny day, were used to focus light on to something the size of what you'd put into a microwave oven?



      There are a few more subtle differences. For example dangerous powers of visible light are so bright you'll surely close your eyes. Furthermore, visible light penetrates less deeply, so you're likely to feel the heat and move away before it does any more than superficial damage to your skin, like a sunburn. On the other hand, microwave radiation is invisible and penetrates more deeply, so you may suffer irreversible injury before even noticing the hazard. The cornea is especially prone to microwave injury since there's no protective reflex to protect it, it has little thermal mass and thus heats quickly, and there's little blood flow to cool it.






      share|cite|improve this answer























      • Of course, solar ovens are a thing - and indeed, to get a decent oven, a square meter of mirrors concentrating the sunlight to the center (with the food item) is more than enough.
        – Luaan
        4 hours ago



















      0














      The trick with microwaves is that they use resonance frequencies of water. Some microwaves can be tuned to meat, vegetables or fish, since the resonance frequency can change slightly in composition. Visible light does not resonate with anything in our bodies.



      Our generally photo-sensitive skin mostly reacts to UV-Ranges, even in a not yet ionizing spectrum. (The wavelength also determines how deep the light can enter the skin!)



      With enough energy, visible light could be dangerous, but the energy needed is far greater without any resonance effects. (think of a child swing)






      share|cite|improve this answer








      New contributor




      Anonymous is a new contributor to this site. Take care in asking for clarification, commenting, and answering.
      Check out our Code of Conduct.


















      • Welcome to Physics SE! Hope you'll enjoy it here. Great first answer.
        – Andrea
        yesterday






      • 9




        Microwaves don't use resonance; it's a common misconception. See explanation e.g. here or anywhere on the Internet.
        – user27542
        yesterday










      • In what is fish water different from meat water or vegetable water? Not to mention that you can heat up butter just fine in a microwave.
        – Dmitry Grigoryev
        yesterday












      • @user27542 Interesting fact, didn't know. Why does visible light not do any dielectric heating? Maybe it's not resonance in the strict sense but you must at least be in the frequency ballpark?
        – Peter A. Schneider
        yesterday








      • 1




        @DmitryGrigoryev -- butter has a lot of water in it. When you put butter in a hot pan, the bubbles are the water boiling off.
        – Pete Becker
        yesterday





















      0














      When we say that microwaves are less energetic, we are talking about the energy in a single photon. The number of photons is also important.



      A single microwave photon is utterly harmless. Its only effect is heat, and it takes a serious amount of heat to damage us. But enough heat, in any form, will kill.



      Visible light has enough energy that single photons can cause chemical reactions, but only in sensitive compounds. That is what happens in our eyes. The chemicals in our eyes are carefully constructed to be extra sensitive to light and that is what makes light visible.



      Ultraviolet light is worse. Here the photons carry enough energy to cause unwanted chemical reactions in most organic compounds. Sun burn and skin cancer results.



      Gamma ray photons from radioactivity are even worse, but they are fortunately rare.






      share|cite|improve this answer





























        0














        There is nothing inherently more or less dangerous about microwaves. Yes, the type of damage via microwave vs. visible light vs. xray are different but weather or not the light cause damage has the same factor for all the spectrum of light - intensity.



        Remember, modern computer-using humans constantly volunteer to be bather in microwaves. Wifi uses exactly the same frequency as microwave ovens. The difference between wifi and microwave ovens is the wattage - the amount of power used to generate the light - the intensity.



        You can cook with visible light if you pump enough power into it - or somehow concentrate it. This is how sun ovens work and how you can burn paper with a magnifying glass. You can also cook with xrays if it is intense enough.



        Side note: Most people don't realise this but high intensity infrared light can blind you as surely as staring at the sun. Just because it is invisible to your eyes does not mean the photons does not hit your retina. High intensity infrared spotlights are sold as part of security systems for infrared sensitive cameras (night vision).






        share|cite|improve this answer





















        • "High intensity" is relative, of course; most spotlights used for "night vision cameras" are still just a few watts, which is rather small compared to sunlight. There is a danger mainly because you don't realize you're too close to the light (and there's no eye response to the increasing amount of light hitting your eyes), and if it's dark, your pupils are maximally dilated. You don't see microwaves, but above about 20 W, the heat is quite noticeable, and a 200 W light is unmistakably warm (those are usually used for heating, though, not IR camera illumination).
          – Luaan
          4 hours ago











        Your Answer





        StackExchange.ifUsing("editor", function () {
        return StackExchange.using("mathjaxEditing", function () {
        StackExchange.MarkdownEditor.creationCallbacks.add(function (editor, postfix) {
        StackExchange.mathjaxEditing.prepareWmdForMathJax(editor, postfix, [["$", "$"], ["\\(","\\)"]]);
        });
        });
        }, "mathjax-editing");

        StackExchange.ready(function() {
        var channelOptions = {
        tags: "".split(" "),
        id: "151"
        };
        initTagRenderer("".split(" "), "".split(" "), channelOptions);

        StackExchange.using("externalEditor", function() {
        // Have to fire editor after snippets, if snippets enabled
        if (StackExchange.settings.snippets.snippetsEnabled) {
        StackExchange.using("snippets", function() {
        createEditor();
        });
        }
        else {
        createEditor();
        }
        });

        function createEditor() {
        StackExchange.prepareEditor({
        heartbeatType: 'answer',
        autoActivateHeartbeat: false,
        convertImagesToLinks: false,
        noModals: true,
        showLowRepImageUploadWarning: true,
        reputationToPostImages: null,
        bindNavPrevention: true,
        postfix: "",
        imageUploader: {
        brandingHtml: "Powered by u003ca class="icon-imgur-white" href="https://imgur.com/"u003eu003c/au003e",
        contentPolicyHtml: "User contributions licensed under u003ca href="https://creativecommons.org/licenses/by-sa/3.0/"u003ecc by-sa 3.0 with attribution requiredu003c/au003e u003ca href="https://stackoverflow.com/legal/content-policy"u003e(content policy)u003c/au003e",
        allowUrls: true
        },
        noCode: true, onDemand: true,
        discardSelector: ".discard-answer"
        ,immediatelyShowMarkdownHelp:true
        });


        }
        });














        draft saved

        draft discarded


















        StackExchange.ready(
        function () {
        StackExchange.openid.initPostLogin('.new-post-login', 'https%3a%2f%2fphysics.stackexchange.com%2fquestions%2f452592%2fif-visible-light-has-more-energy-than-microwaves-why-isnt-visible-light-danger%23new-answer', 'question_page');
        }
        );

        Post as a guest















        Required, but never shown

























        10 Answers
        10






        active

        oldest

        votes








        10 Answers
        10






        active

        oldest

        votes









        active

        oldest

        votes






        active

        oldest

        votes









        55














        Your question contains a premise that is false: Microwaves do not have less energy than visible light per se. They only have less energy per photon, as per the Planck--Einstein relation, $E = hf$. In other words, you can raise the power of electromagnetic radiation to a dangerous level at any wavelength, if only you generate enough photons -- as your microwave oven does.



        That very much includes visible light. You can easily verify this by waiting for a sunny day, getting out your magnifying glass, and using it to focus sunlight onto your forearm. (Just be sure to stop when you start noticing the smell of burning hair.) In conclusion, then, sunlight is dangerous!



        EDIT: As user JMac suggests in his comment below, better focus the sunlight on a piece of paper first. Watch it char and maybe even burn, and then think twice before trying it on your arm. Also on the subject of offering words of caution, make sure there's nothing around that piece of paper that can burn.






        share|cite|improve this answer










        New contributor




        Thomas Blankenhorn is a new contributor to this site. Take care in asking for clarification, commenting, and answering.
        Check out our Code of Conduct.














        • 12




          "You can easily verify this by waiting for a sunny day, getting out your magnifying glass, and using it to focus sunlight onto your forearm." Nope. Don't do this. It's not good for you, unless you like burns, sunburn and melanoma.
          – wizzwizz4
          yesterday






        • 7




          I was half-joking, and wizzwizz4 makes a fair point. While the experiment does demonstrate that visible light can be harmful, the flip side is, don't do it if you don't like pain! No to the specific point about sunburn and melanoma, though. These are both caused by ionizing radiation (ultraviolet and shorter wavelengths),. Most of that gets filtered out by the glass (or plastic) that lenses are made of. During the few seconds until one feels the burn, twitches, and thereby ends the experiment, one won't accumulate enough to cause lasting harm.
          – Thomas Blankenhorn
          yesterday








        • 2




          Didn't know that plastic lenses also filtered out UV. (I assume it's only some plastics.) Thanks for the information!
          – wizzwizz4
          yesterday






        • 20




          Safety comment - For any readers who may not realize this - The note about burning hair is not a joke. If you focus sunlight on your skin with a magnifying glass, you will burn yourself severely. DO NOT DO THIS.
          – TypeIA
          23 hours ago






        • 6




          You may want to suggest that they try the experiment on something like a piece of paper first. You can quickly figure out a good reason on your own for why you wouldn't want to focus it on your arm.
          – JMac
          21 hours ago
















        55














        Your question contains a premise that is false: Microwaves do not have less energy than visible light per se. They only have less energy per photon, as per the Planck--Einstein relation, $E = hf$. In other words, you can raise the power of electromagnetic radiation to a dangerous level at any wavelength, if only you generate enough photons -- as your microwave oven does.



        That very much includes visible light. You can easily verify this by waiting for a sunny day, getting out your magnifying glass, and using it to focus sunlight onto your forearm. (Just be sure to stop when you start noticing the smell of burning hair.) In conclusion, then, sunlight is dangerous!



        EDIT: As user JMac suggests in his comment below, better focus the sunlight on a piece of paper first. Watch it char and maybe even burn, and then think twice before trying it on your arm. Also on the subject of offering words of caution, make sure there's nothing around that piece of paper that can burn.






        share|cite|improve this answer










        New contributor




        Thomas Blankenhorn is a new contributor to this site. Take care in asking for clarification, commenting, and answering.
        Check out our Code of Conduct.














        • 12




          "You can easily verify this by waiting for a sunny day, getting out your magnifying glass, and using it to focus sunlight onto your forearm." Nope. Don't do this. It's not good for you, unless you like burns, sunburn and melanoma.
          – wizzwizz4
          yesterday






        • 7




          I was half-joking, and wizzwizz4 makes a fair point. While the experiment does demonstrate that visible light can be harmful, the flip side is, don't do it if you don't like pain! No to the specific point about sunburn and melanoma, though. These are both caused by ionizing radiation (ultraviolet and shorter wavelengths),. Most of that gets filtered out by the glass (or plastic) that lenses are made of. During the few seconds until one feels the burn, twitches, and thereby ends the experiment, one won't accumulate enough to cause lasting harm.
          – Thomas Blankenhorn
          yesterday








        • 2




          Didn't know that plastic lenses also filtered out UV. (I assume it's only some plastics.) Thanks for the information!
          – wizzwizz4
          yesterday






        • 20




          Safety comment - For any readers who may not realize this - The note about burning hair is not a joke. If you focus sunlight on your skin with a magnifying glass, you will burn yourself severely. DO NOT DO THIS.
          – TypeIA
          23 hours ago






        • 6




          You may want to suggest that they try the experiment on something like a piece of paper first. You can quickly figure out a good reason on your own for why you wouldn't want to focus it on your arm.
          – JMac
          21 hours ago














        55












        55








        55






        Your question contains a premise that is false: Microwaves do not have less energy than visible light per se. They only have less energy per photon, as per the Planck--Einstein relation, $E = hf$. In other words, you can raise the power of electromagnetic radiation to a dangerous level at any wavelength, if only you generate enough photons -- as your microwave oven does.



        That very much includes visible light. You can easily verify this by waiting for a sunny day, getting out your magnifying glass, and using it to focus sunlight onto your forearm. (Just be sure to stop when you start noticing the smell of burning hair.) In conclusion, then, sunlight is dangerous!



        EDIT: As user JMac suggests in his comment below, better focus the sunlight on a piece of paper first. Watch it char and maybe even burn, and then think twice before trying it on your arm. Also on the subject of offering words of caution, make sure there's nothing around that piece of paper that can burn.






        share|cite|improve this answer










        New contributor




        Thomas Blankenhorn is a new contributor to this site. Take care in asking for clarification, commenting, and answering.
        Check out our Code of Conduct.









        Your question contains a premise that is false: Microwaves do not have less energy than visible light per se. They only have less energy per photon, as per the Planck--Einstein relation, $E = hf$. In other words, you can raise the power of electromagnetic radiation to a dangerous level at any wavelength, if only you generate enough photons -- as your microwave oven does.



        That very much includes visible light. You can easily verify this by waiting for a sunny day, getting out your magnifying glass, and using it to focus sunlight onto your forearm. (Just be sure to stop when you start noticing the smell of burning hair.) In conclusion, then, sunlight is dangerous!



        EDIT: As user JMac suggests in his comment below, better focus the sunlight on a piece of paper first. Watch it char and maybe even burn, and then think twice before trying it on your arm. Also on the subject of offering words of caution, make sure there's nothing around that piece of paper that can burn.







        share|cite|improve this answer










        New contributor




        Thomas Blankenhorn is a new contributor to this site. Take care in asking for clarification, commenting, and answering.
        Check out our Code of Conduct.









        share|cite|improve this answer



        share|cite|improve this answer








        edited 8 mins ago









        John Rennie

        271k43533782




        271k43533782






        New contributor




        Thomas Blankenhorn is a new contributor to this site. Take care in asking for clarification, commenting, and answering.
        Check out our Code of Conduct.









        answered yesterday









        Thomas BlankenhornThomas Blankenhorn

        53127




        53127




        New contributor




        Thomas Blankenhorn is a new contributor to this site. Take care in asking for clarification, commenting, and answering.
        Check out our Code of Conduct.





        New contributor





        Thomas Blankenhorn is a new contributor to this site. Take care in asking for clarification, commenting, and answering.
        Check out our Code of Conduct.






        Thomas Blankenhorn is a new contributor to this site. Take care in asking for clarification, commenting, and answering.
        Check out our Code of Conduct.








        • 12




          "You can easily verify this by waiting for a sunny day, getting out your magnifying glass, and using it to focus sunlight onto your forearm." Nope. Don't do this. It's not good for you, unless you like burns, sunburn and melanoma.
          – wizzwizz4
          yesterday






        • 7




          I was half-joking, and wizzwizz4 makes a fair point. While the experiment does demonstrate that visible light can be harmful, the flip side is, don't do it if you don't like pain! No to the specific point about sunburn and melanoma, though. These are both caused by ionizing radiation (ultraviolet and shorter wavelengths),. Most of that gets filtered out by the glass (or plastic) that lenses are made of. During the few seconds until one feels the burn, twitches, and thereby ends the experiment, one won't accumulate enough to cause lasting harm.
          – Thomas Blankenhorn
          yesterday








        • 2




          Didn't know that plastic lenses also filtered out UV. (I assume it's only some plastics.) Thanks for the information!
          – wizzwizz4
          yesterday






        • 20




          Safety comment - For any readers who may not realize this - The note about burning hair is not a joke. If you focus sunlight on your skin with a magnifying glass, you will burn yourself severely. DO NOT DO THIS.
          – TypeIA
          23 hours ago






        • 6




          You may want to suggest that they try the experiment on something like a piece of paper first. You can quickly figure out a good reason on your own for why you wouldn't want to focus it on your arm.
          – JMac
          21 hours ago














        • 12




          "You can easily verify this by waiting for a sunny day, getting out your magnifying glass, and using it to focus sunlight onto your forearm." Nope. Don't do this. It's not good for you, unless you like burns, sunburn and melanoma.
          – wizzwizz4
          yesterday






        • 7




          I was half-joking, and wizzwizz4 makes a fair point. While the experiment does demonstrate that visible light can be harmful, the flip side is, don't do it if you don't like pain! No to the specific point about sunburn and melanoma, though. These are both caused by ionizing radiation (ultraviolet and shorter wavelengths),. Most of that gets filtered out by the glass (or plastic) that lenses are made of. During the few seconds until one feels the burn, twitches, and thereby ends the experiment, one won't accumulate enough to cause lasting harm.
          – Thomas Blankenhorn
          yesterday








        • 2




          Didn't know that plastic lenses also filtered out UV. (I assume it's only some plastics.) Thanks for the information!
          – wizzwizz4
          yesterday






        • 20




          Safety comment - For any readers who may not realize this - The note about burning hair is not a joke. If you focus sunlight on your skin with a magnifying glass, you will burn yourself severely. DO NOT DO THIS.
          – TypeIA
          23 hours ago






        • 6




          You may want to suggest that they try the experiment on something like a piece of paper first. You can quickly figure out a good reason on your own for why you wouldn't want to focus it on your arm.
          – JMac
          21 hours ago








        12




        12




        "You can easily verify this by waiting for a sunny day, getting out your magnifying glass, and using it to focus sunlight onto your forearm." Nope. Don't do this. It's not good for you, unless you like burns, sunburn and melanoma.
        – wizzwizz4
        yesterday




        "You can easily verify this by waiting for a sunny day, getting out your magnifying glass, and using it to focus sunlight onto your forearm." Nope. Don't do this. It's not good for you, unless you like burns, sunburn and melanoma.
        – wizzwizz4
        yesterday




        7




        7




        I was half-joking, and wizzwizz4 makes a fair point. While the experiment does demonstrate that visible light can be harmful, the flip side is, don't do it if you don't like pain! No to the specific point about sunburn and melanoma, though. These are both caused by ionizing radiation (ultraviolet and shorter wavelengths),. Most of that gets filtered out by the glass (or plastic) that lenses are made of. During the few seconds until one feels the burn, twitches, and thereby ends the experiment, one won't accumulate enough to cause lasting harm.
        – Thomas Blankenhorn
        yesterday






        I was half-joking, and wizzwizz4 makes a fair point. While the experiment does demonstrate that visible light can be harmful, the flip side is, don't do it if you don't like pain! No to the specific point about sunburn and melanoma, though. These are both caused by ionizing radiation (ultraviolet and shorter wavelengths),. Most of that gets filtered out by the glass (or plastic) that lenses are made of. During the few seconds until one feels the burn, twitches, and thereby ends the experiment, one won't accumulate enough to cause lasting harm.
        – Thomas Blankenhorn
        yesterday






        2




        2




        Didn't know that plastic lenses also filtered out UV. (I assume it's only some plastics.) Thanks for the information!
        – wizzwizz4
        yesterday




        Didn't know that plastic lenses also filtered out UV. (I assume it's only some plastics.) Thanks for the information!
        – wizzwizz4
        yesterday




        20




        20




        Safety comment - For any readers who may not realize this - The note about burning hair is not a joke. If you focus sunlight on your skin with a magnifying glass, you will burn yourself severely. DO NOT DO THIS.
        – TypeIA
        23 hours ago




        Safety comment - For any readers who may not realize this - The note about burning hair is not a joke. If you focus sunlight on your skin with a magnifying glass, you will burn yourself severely. DO NOT DO THIS.
        – TypeIA
        23 hours ago




        6




        6




        You may want to suggest that they try the experiment on something like a piece of paper first. You can quickly figure out a good reason on your own for why you wouldn't want to focus it on your arm.
        – JMac
        21 hours ago




        You may want to suggest that they try the experiment on something like a piece of paper first. You can quickly figure out a good reason on your own for why you wouldn't want to focus it on your arm.
        – JMac
        21 hours ago











        41














        If you stare at the Sun you’ll go blind. And if you spend a lot of time in the sun, you’re likely to get skin cancers. So visible light seems plenty dangerous to me.



        Some of the damage may actually be from infrared and ultraviolet light, but these are close in frequency to visible light and very far from microwaves.



        By the way, the intensity also matters, not just the frequency. In terms of photons, it matters not only how energetic each photon is, but also how many photons are arriving per second.






        share|cite|improve this answer

















        • 16




          not to forget laser guns,
          – anna v
          yesterday






        • 22




          This answer could be improved by addressing the OP's comparison with microwaves and discussing ways in which microwaves are dangerous (heating) and are not (cancer).
          – Ben Crowell
          yesterday






        • 4




          Could also compare the wattage of a microwave vs sunlight
          – JollyJoker
          yesterday








        • 6




          @BenCrowell The thing is, there's nothing really dangerous about microwaves - the only damage they do is through heating, which doesn't depend on the wavelength as long as the material absorbs most of the energy anyway. The same amount of energy from visible light does far more damage. But adding it to the answer is probably a good idea, since it seems suse doesn't understand that part.
          – Luaan
          yesterday






        • 10




          "And if you spend a lot of time in the sun, you’re likely to get skin cancers. So visible light seems plenty dangerous to me." - Skin cancer is caused by the UV radiation the sun emits, not visible light.
          – marcelm
          yesterday
















        41














        If you stare at the Sun you’ll go blind. And if you spend a lot of time in the sun, you’re likely to get skin cancers. So visible light seems plenty dangerous to me.



        Some of the damage may actually be from infrared and ultraviolet light, but these are close in frequency to visible light and very far from microwaves.



        By the way, the intensity also matters, not just the frequency. In terms of photons, it matters not only how energetic each photon is, but also how many photons are arriving per second.






        share|cite|improve this answer

















        • 16




          not to forget laser guns,
          – anna v
          yesterday






        • 22




          This answer could be improved by addressing the OP's comparison with microwaves and discussing ways in which microwaves are dangerous (heating) and are not (cancer).
          – Ben Crowell
          yesterday






        • 4




          Could also compare the wattage of a microwave vs sunlight
          – JollyJoker
          yesterday








        • 6




          @BenCrowell The thing is, there's nothing really dangerous about microwaves - the only damage they do is through heating, which doesn't depend on the wavelength as long as the material absorbs most of the energy anyway. The same amount of energy from visible light does far more damage. But adding it to the answer is probably a good idea, since it seems suse doesn't understand that part.
          – Luaan
          yesterday






        • 10




          "And if you spend a lot of time in the sun, you’re likely to get skin cancers. So visible light seems plenty dangerous to me." - Skin cancer is caused by the UV radiation the sun emits, not visible light.
          – marcelm
          yesterday














        41












        41








        41






        If you stare at the Sun you’ll go blind. And if you spend a lot of time in the sun, you’re likely to get skin cancers. So visible light seems plenty dangerous to me.



        Some of the damage may actually be from infrared and ultraviolet light, but these are close in frequency to visible light and very far from microwaves.



        By the way, the intensity also matters, not just the frequency. In terms of photons, it matters not only how energetic each photon is, but also how many photons are arriving per second.






        share|cite|improve this answer












        If you stare at the Sun you’ll go blind. And if you spend a lot of time in the sun, you’re likely to get skin cancers. So visible light seems plenty dangerous to me.



        Some of the damage may actually be from infrared and ultraviolet light, but these are close in frequency to visible light and very far from microwaves.



        By the way, the intensity also matters, not just the frequency. In terms of photons, it matters not only how energetic each photon is, but also how many photons are arriving per second.







        share|cite|improve this answer












        share|cite|improve this answer



        share|cite|improve this answer










        answered yesterday









        G. SmithG. Smith

        4,8751021




        4,8751021








        • 16




          not to forget laser guns,
          – anna v
          yesterday






        • 22




          This answer could be improved by addressing the OP's comparison with microwaves and discussing ways in which microwaves are dangerous (heating) and are not (cancer).
          – Ben Crowell
          yesterday






        • 4




          Could also compare the wattage of a microwave vs sunlight
          – JollyJoker
          yesterday








        • 6




          @BenCrowell The thing is, there's nothing really dangerous about microwaves - the only damage they do is through heating, which doesn't depend on the wavelength as long as the material absorbs most of the energy anyway. The same amount of energy from visible light does far more damage. But adding it to the answer is probably a good idea, since it seems suse doesn't understand that part.
          – Luaan
          yesterday






        • 10




          "And if you spend a lot of time in the sun, you’re likely to get skin cancers. So visible light seems plenty dangerous to me." - Skin cancer is caused by the UV radiation the sun emits, not visible light.
          – marcelm
          yesterday














        • 16




          not to forget laser guns,
          – anna v
          yesterday






        • 22




          This answer could be improved by addressing the OP's comparison with microwaves and discussing ways in which microwaves are dangerous (heating) and are not (cancer).
          – Ben Crowell
          yesterday






        • 4




          Could also compare the wattage of a microwave vs sunlight
          – JollyJoker
          yesterday








        • 6




          @BenCrowell The thing is, there's nothing really dangerous about microwaves - the only damage they do is through heating, which doesn't depend on the wavelength as long as the material absorbs most of the energy anyway. The same amount of energy from visible light does far more damage. But adding it to the answer is probably a good idea, since it seems suse doesn't understand that part.
          – Luaan
          yesterday






        • 10




          "And if you spend a lot of time in the sun, you’re likely to get skin cancers. So visible light seems plenty dangerous to me." - Skin cancer is caused by the UV radiation the sun emits, not visible light.
          – marcelm
          yesterday








        16




        16




        not to forget laser guns,
        – anna v
        yesterday




        not to forget laser guns,
        – anna v
        yesterday




        22




        22




        This answer could be improved by addressing the OP's comparison with microwaves and discussing ways in which microwaves are dangerous (heating) and are not (cancer).
        – Ben Crowell
        yesterday




        This answer could be improved by addressing the OP's comparison with microwaves and discussing ways in which microwaves are dangerous (heating) and are not (cancer).
        – Ben Crowell
        yesterday




        4




        4




        Could also compare the wattage of a microwave vs sunlight
        – JollyJoker
        yesterday






        Could also compare the wattage of a microwave vs sunlight
        – JollyJoker
        yesterday






        6




        6




        @BenCrowell The thing is, there's nothing really dangerous about microwaves - the only damage they do is through heating, which doesn't depend on the wavelength as long as the material absorbs most of the energy anyway. The same amount of energy from visible light does far more damage. But adding it to the answer is probably a good idea, since it seems suse doesn't understand that part.
        – Luaan
        yesterday




        @BenCrowell The thing is, there's nothing really dangerous about microwaves - the only damage they do is through heating, which doesn't depend on the wavelength as long as the material absorbs most of the energy anyway. The same amount of energy from visible light does far more damage. But adding it to the answer is probably a good idea, since it seems suse doesn't understand that part.
        – Luaan
        yesterday




        10




        10




        "And if you spend a lot of time in the sun, you’re likely to get skin cancers. So visible light seems plenty dangerous to me." - Skin cancer is caused by the UV radiation the sun emits, not visible light.
        – marcelm
        yesterday




        "And if you spend a lot of time in the sun, you’re likely to get skin cancers. So visible light seems plenty dangerous to me." - Skin cancer is caused by the UV radiation the sun emits, not visible light.
        – marcelm
        yesterday











        32














        The dose (or, in this case, the intensity) makes the poison. You're constantly exposed to microwaves since that's what 99% of wireless communication devices use, and you're also constantly exposed to visible light unless you sleep in an isolation tank. Both can be dangerous if you increase the intensity sufficiently.






        share|cite|improve this answer

















        • 1




          For that matter, you are constantly exposed to microwaves from the sun and, for that matter, the CMB. Just at very low levels.
          – WhatRoughBeast
          23 hours ago






        • 1




          Let me put some numbers behind that. A standard microwave oven puts 650W of electromagnetic energy into whatever you put inside it (many ovens actually do more these days). A wifi router is limited to 4 watts EIRP of RF radiation, which falls off with distance on the familiar inverse-square law. A modern bicycle headlamp, operating from a 3-watt dynamo, uses LEDs with a thermal efficiency of about 50%, so emits about 1.5 watts of visible light energy - and that's pretty bright. Now imagine looking at 650W of visible light…
          – Chromatix
          2 hours ago
















        32














        The dose (or, in this case, the intensity) makes the poison. You're constantly exposed to microwaves since that's what 99% of wireless communication devices use, and you're also constantly exposed to visible light unless you sleep in an isolation tank. Both can be dangerous if you increase the intensity sufficiently.






        share|cite|improve this answer

















        • 1




          For that matter, you are constantly exposed to microwaves from the sun and, for that matter, the CMB. Just at very low levels.
          – WhatRoughBeast
          23 hours ago






        • 1




          Let me put some numbers behind that. A standard microwave oven puts 650W of electromagnetic energy into whatever you put inside it (many ovens actually do more these days). A wifi router is limited to 4 watts EIRP of RF radiation, which falls off with distance on the familiar inverse-square law. A modern bicycle headlamp, operating from a 3-watt dynamo, uses LEDs with a thermal efficiency of about 50%, so emits about 1.5 watts of visible light energy - and that's pretty bright. Now imagine looking at 650W of visible light…
          – Chromatix
          2 hours ago














        32












        32








        32






        The dose (or, in this case, the intensity) makes the poison. You're constantly exposed to microwaves since that's what 99% of wireless communication devices use, and you're also constantly exposed to visible light unless you sleep in an isolation tank. Both can be dangerous if you increase the intensity sufficiently.






        share|cite|improve this answer












        The dose (or, in this case, the intensity) makes the poison. You're constantly exposed to microwaves since that's what 99% of wireless communication devices use, and you're also constantly exposed to visible light unless you sleep in an isolation tank. Both can be dangerous if you increase the intensity sufficiently.







        share|cite|improve this answer












        share|cite|improve this answer



        share|cite|improve this answer










        answered yesterday









        Dmitry GrigoryevDmitry Grigoryev

        2,8181624




        2,8181624








        • 1




          For that matter, you are constantly exposed to microwaves from the sun and, for that matter, the CMB. Just at very low levels.
          – WhatRoughBeast
          23 hours ago






        • 1




          Let me put some numbers behind that. A standard microwave oven puts 650W of electromagnetic energy into whatever you put inside it (many ovens actually do more these days). A wifi router is limited to 4 watts EIRP of RF radiation, which falls off with distance on the familiar inverse-square law. A modern bicycle headlamp, operating from a 3-watt dynamo, uses LEDs with a thermal efficiency of about 50%, so emits about 1.5 watts of visible light energy - and that's pretty bright. Now imagine looking at 650W of visible light…
          – Chromatix
          2 hours ago














        • 1




          For that matter, you are constantly exposed to microwaves from the sun and, for that matter, the CMB. Just at very low levels.
          – WhatRoughBeast
          23 hours ago






        • 1




          Let me put some numbers behind that. A standard microwave oven puts 650W of electromagnetic energy into whatever you put inside it (many ovens actually do more these days). A wifi router is limited to 4 watts EIRP of RF radiation, which falls off with distance on the familiar inverse-square law. A modern bicycle headlamp, operating from a 3-watt dynamo, uses LEDs with a thermal efficiency of about 50%, so emits about 1.5 watts of visible light energy - and that's pretty bright. Now imagine looking at 650W of visible light…
          – Chromatix
          2 hours ago








        1




        1




        For that matter, you are constantly exposed to microwaves from the sun and, for that matter, the CMB. Just at very low levels.
        – WhatRoughBeast
        23 hours ago




        For that matter, you are constantly exposed to microwaves from the sun and, for that matter, the CMB. Just at very low levels.
        – WhatRoughBeast
        23 hours ago




        1




        1




        Let me put some numbers behind that. A standard microwave oven puts 650W of electromagnetic energy into whatever you put inside it (many ovens actually do more these days). A wifi router is limited to 4 watts EIRP of RF radiation, which falls off with distance on the familiar inverse-square law. A modern bicycle headlamp, operating from a 3-watt dynamo, uses LEDs with a thermal efficiency of about 50%, so emits about 1.5 watts of visible light energy - and that's pretty bright. Now imagine looking at 650W of visible light…
        – Chromatix
        2 hours ago




        Let me put some numbers behind that. A standard microwave oven puts 650W of electromagnetic energy into whatever you put inside it (many ovens actually do more these days). A wifi router is limited to 4 watts EIRP of RF radiation, which falls off with distance on the familiar inverse-square law. A modern bicycle headlamp, operating from a 3-watt dynamo, uses LEDs with a thermal efficiency of about 50%, so emits about 1.5 watts of visible light energy - and that's pretty bright. Now imagine looking at 650W of visible light…
        – Chromatix
        2 hours ago











        16














        Assuming the intensity is the same, microwaves are more dangerous than visible light because they penetrate the skin to a greater depth (1-2 cm; more info is in Wikipedia).



        Humans have more adaptation to visible light than to microwave radiation, because they were exposed to light for millions of years. This is expressed in two ways:




        • Proteins in the epidermis (outer skin layer) are more resistant to heat than those in the deeper skin layers

        • There are more nerve endings in the outer layers of skin, so dangerous heating by visible light causes more pain, urging you to escape the dangerous situation


        Oh, and the most obvious difference: visible light is visible. Dangerous levels of visible light (e.g. in a solar cooker), to our eyes, look blinding and obviously dangerous. Dangerous levels of microwave radiation are invisible.






        share|cite|improve this answer























        • In general, anything that penetrates more is less dangerous; since it means it's less ionising. For example, gamma radiation; when it does ionise does a whole lot of damage... however it's such high energy, most of them are able to pass through the body without impacting it at all. While something like an alpha particle (sure, it's not EM, but concept applies) becomes VERY dangerous because it doesn't penetrate at all. Obviously, reflecting and passing through are the same in that the energy isn't absorbed by the organs; meaning no ionisation occurs.
          – UKMonkey
          yesterday








        • 1




          Wikipedia doesn't seem to support your claim. For instance, the microwave intensity found to cause cataracts in rabbits is 150 mW/cm2 for 100 minutes. That is roughly equal to the intensity of natural sunlight. I believe staring at the Sun will make you blind much faster.
          – Dmitry Grigoryev
          yesterday






        • 2




          @UKMonkey Re "alpha is more dangerous than gamma because it penetrates less": Simply not true: "exposure to most alpha particles originating outside the body is not a serious hazard." Or maybe a Briton would trust the BBC. The same applies to light: The skin absorbs/reflects lots of it and protects underlying tissue; while microwaves can penetrate it and reach living cells.
          – Peter A. Schneider
          yesterday












        • @PeterA.Schneider Sure - emphasis on outside the body. Inside the body is a very different story. That's because our skin has been very well adapted to protect us. The point is that you can't just say "it's more penetrating -> it's more dangerous" EM Radiation isn't a gun - if it goes through you, it's harmless. Now there's a sweet spot where all the radiation is absorbed in the body, and it penetrates some distance - which is how cancer treatment works; however attempting to suggest that masters in physics is related to the media is insulting, and I'd be thankful if you could refrain.
          – UKMonkey
          yesterday










        • @UKMonkey I didn't mean to insult, and I didn't know you had a Master's degree in Physics.-- "Outside the body": Well, that's the setting of the question: Obviously the radiation (light or microwave) is meant to originate outside the body, and the main reason light does less harm is that it doesn't penetrate the skin much.
          – Peter A. Schneider
          yesterday


















        16














        Assuming the intensity is the same, microwaves are more dangerous than visible light because they penetrate the skin to a greater depth (1-2 cm; more info is in Wikipedia).



        Humans have more adaptation to visible light than to microwave radiation, because they were exposed to light for millions of years. This is expressed in two ways:




        • Proteins in the epidermis (outer skin layer) are more resistant to heat than those in the deeper skin layers

        • There are more nerve endings in the outer layers of skin, so dangerous heating by visible light causes more pain, urging you to escape the dangerous situation


        Oh, and the most obvious difference: visible light is visible. Dangerous levels of visible light (e.g. in a solar cooker), to our eyes, look blinding and obviously dangerous. Dangerous levels of microwave radiation are invisible.






        share|cite|improve this answer























        • In general, anything that penetrates more is less dangerous; since it means it's less ionising. For example, gamma radiation; when it does ionise does a whole lot of damage... however it's such high energy, most of them are able to pass through the body without impacting it at all. While something like an alpha particle (sure, it's not EM, but concept applies) becomes VERY dangerous because it doesn't penetrate at all. Obviously, reflecting and passing through are the same in that the energy isn't absorbed by the organs; meaning no ionisation occurs.
          – UKMonkey
          yesterday








        • 1




          Wikipedia doesn't seem to support your claim. For instance, the microwave intensity found to cause cataracts in rabbits is 150 mW/cm2 for 100 minutes. That is roughly equal to the intensity of natural sunlight. I believe staring at the Sun will make you blind much faster.
          – Dmitry Grigoryev
          yesterday






        • 2




          @UKMonkey Re "alpha is more dangerous than gamma because it penetrates less": Simply not true: "exposure to most alpha particles originating outside the body is not a serious hazard." Or maybe a Briton would trust the BBC. The same applies to light: The skin absorbs/reflects lots of it and protects underlying tissue; while microwaves can penetrate it and reach living cells.
          – Peter A. Schneider
          yesterday












        • @PeterA.Schneider Sure - emphasis on outside the body. Inside the body is a very different story. That's because our skin has been very well adapted to protect us. The point is that you can't just say "it's more penetrating -> it's more dangerous" EM Radiation isn't a gun - if it goes through you, it's harmless. Now there's a sweet spot where all the radiation is absorbed in the body, and it penetrates some distance - which is how cancer treatment works; however attempting to suggest that masters in physics is related to the media is insulting, and I'd be thankful if you could refrain.
          – UKMonkey
          yesterday










        • @UKMonkey I didn't mean to insult, and I didn't know you had a Master's degree in Physics.-- "Outside the body": Well, that's the setting of the question: Obviously the radiation (light or microwave) is meant to originate outside the body, and the main reason light does less harm is that it doesn't penetrate the skin much.
          – Peter A. Schneider
          yesterday
















        16












        16








        16






        Assuming the intensity is the same, microwaves are more dangerous than visible light because they penetrate the skin to a greater depth (1-2 cm; more info is in Wikipedia).



        Humans have more adaptation to visible light than to microwave radiation, because they were exposed to light for millions of years. This is expressed in two ways:




        • Proteins in the epidermis (outer skin layer) are more resistant to heat than those in the deeper skin layers

        • There are more nerve endings in the outer layers of skin, so dangerous heating by visible light causes more pain, urging you to escape the dangerous situation


        Oh, and the most obvious difference: visible light is visible. Dangerous levels of visible light (e.g. in a solar cooker), to our eyes, look blinding and obviously dangerous. Dangerous levels of microwave radiation are invisible.






        share|cite|improve this answer














        Assuming the intensity is the same, microwaves are more dangerous than visible light because they penetrate the skin to a greater depth (1-2 cm; more info is in Wikipedia).



        Humans have more adaptation to visible light than to microwave radiation, because they were exposed to light for millions of years. This is expressed in two ways:




        • Proteins in the epidermis (outer skin layer) are more resistant to heat than those in the deeper skin layers

        • There are more nerve endings in the outer layers of skin, so dangerous heating by visible light causes more pain, urging you to escape the dangerous situation


        Oh, and the most obvious difference: visible light is visible. Dangerous levels of visible light (e.g. in a solar cooker), to our eyes, look blinding and obviously dangerous. Dangerous levels of microwave radiation are invisible.







        share|cite|improve this answer














        share|cite|improve this answer



        share|cite|improve this answer








        edited yesterday

























        answered yesterday









        user27542user27542

        5611415




        5611415












        • In general, anything that penetrates more is less dangerous; since it means it's less ionising. For example, gamma radiation; when it does ionise does a whole lot of damage... however it's such high energy, most of them are able to pass through the body without impacting it at all. While something like an alpha particle (sure, it's not EM, but concept applies) becomes VERY dangerous because it doesn't penetrate at all. Obviously, reflecting and passing through are the same in that the energy isn't absorbed by the organs; meaning no ionisation occurs.
          – UKMonkey
          yesterday








        • 1




          Wikipedia doesn't seem to support your claim. For instance, the microwave intensity found to cause cataracts in rabbits is 150 mW/cm2 for 100 minutes. That is roughly equal to the intensity of natural sunlight. I believe staring at the Sun will make you blind much faster.
          – Dmitry Grigoryev
          yesterday






        • 2




          @UKMonkey Re "alpha is more dangerous than gamma because it penetrates less": Simply not true: "exposure to most alpha particles originating outside the body is not a serious hazard." Or maybe a Briton would trust the BBC. The same applies to light: The skin absorbs/reflects lots of it and protects underlying tissue; while microwaves can penetrate it and reach living cells.
          – Peter A. Schneider
          yesterday












        • @PeterA.Schneider Sure - emphasis on outside the body. Inside the body is a very different story. That's because our skin has been very well adapted to protect us. The point is that you can't just say "it's more penetrating -> it's more dangerous" EM Radiation isn't a gun - if it goes through you, it's harmless. Now there's a sweet spot where all the radiation is absorbed in the body, and it penetrates some distance - which is how cancer treatment works; however attempting to suggest that masters in physics is related to the media is insulting, and I'd be thankful if you could refrain.
          – UKMonkey
          yesterday










        • @UKMonkey I didn't mean to insult, and I didn't know you had a Master's degree in Physics.-- "Outside the body": Well, that's the setting of the question: Obviously the radiation (light or microwave) is meant to originate outside the body, and the main reason light does less harm is that it doesn't penetrate the skin much.
          – Peter A. Schneider
          yesterday




















        • In general, anything that penetrates more is less dangerous; since it means it's less ionising. For example, gamma radiation; when it does ionise does a whole lot of damage... however it's such high energy, most of them are able to pass through the body without impacting it at all. While something like an alpha particle (sure, it's not EM, but concept applies) becomes VERY dangerous because it doesn't penetrate at all. Obviously, reflecting and passing through are the same in that the energy isn't absorbed by the organs; meaning no ionisation occurs.
          – UKMonkey
          yesterday








        • 1




          Wikipedia doesn't seem to support your claim. For instance, the microwave intensity found to cause cataracts in rabbits is 150 mW/cm2 for 100 minutes. That is roughly equal to the intensity of natural sunlight. I believe staring at the Sun will make you blind much faster.
          – Dmitry Grigoryev
          yesterday






        • 2




          @UKMonkey Re "alpha is more dangerous than gamma because it penetrates less": Simply not true: "exposure to most alpha particles originating outside the body is not a serious hazard." Or maybe a Briton would trust the BBC. The same applies to light: The skin absorbs/reflects lots of it and protects underlying tissue; while microwaves can penetrate it and reach living cells.
          – Peter A. Schneider
          yesterday












        • @PeterA.Schneider Sure - emphasis on outside the body. Inside the body is a very different story. That's because our skin has been very well adapted to protect us. The point is that you can't just say "it's more penetrating -> it's more dangerous" EM Radiation isn't a gun - if it goes through you, it's harmless. Now there's a sweet spot where all the radiation is absorbed in the body, and it penetrates some distance - which is how cancer treatment works; however attempting to suggest that masters in physics is related to the media is insulting, and I'd be thankful if you could refrain.
          – UKMonkey
          yesterday










        • @UKMonkey I didn't mean to insult, and I didn't know you had a Master's degree in Physics.-- "Outside the body": Well, that's the setting of the question: Obviously the radiation (light or microwave) is meant to originate outside the body, and the main reason light does less harm is that it doesn't penetrate the skin much.
          – Peter A. Schneider
          yesterday


















        In general, anything that penetrates more is less dangerous; since it means it's less ionising. For example, gamma radiation; when it does ionise does a whole lot of damage... however it's such high energy, most of them are able to pass through the body without impacting it at all. While something like an alpha particle (sure, it's not EM, but concept applies) becomes VERY dangerous because it doesn't penetrate at all. Obviously, reflecting and passing through are the same in that the energy isn't absorbed by the organs; meaning no ionisation occurs.
        – UKMonkey
        yesterday






        In general, anything that penetrates more is less dangerous; since it means it's less ionising. For example, gamma radiation; when it does ionise does a whole lot of damage... however it's such high energy, most of them are able to pass through the body without impacting it at all. While something like an alpha particle (sure, it's not EM, but concept applies) becomes VERY dangerous because it doesn't penetrate at all. Obviously, reflecting and passing through are the same in that the energy isn't absorbed by the organs; meaning no ionisation occurs.
        – UKMonkey
        yesterday






        1




        1




        Wikipedia doesn't seem to support your claim. For instance, the microwave intensity found to cause cataracts in rabbits is 150 mW/cm2 for 100 minutes. That is roughly equal to the intensity of natural sunlight. I believe staring at the Sun will make you blind much faster.
        – Dmitry Grigoryev
        yesterday




        Wikipedia doesn't seem to support your claim. For instance, the microwave intensity found to cause cataracts in rabbits is 150 mW/cm2 for 100 minutes. That is roughly equal to the intensity of natural sunlight. I believe staring at the Sun will make you blind much faster.
        – Dmitry Grigoryev
        yesterday




        2




        2




        @UKMonkey Re "alpha is more dangerous than gamma because it penetrates less": Simply not true: "exposure to most alpha particles originating outside the body is not a serious hazard." Or maybe a Briton would trust the BBC. The same applies to light: The skin absorbs/reflects lots of it and protects underlying tissue; while microwaves can penetrate it and reach living cells.
        – Peter A. Schneider
        yesterday






        @UKMonkey Re "alpha is more dangerous than gamma because it penetrates less": Simply not true: "exposure to most alpha particles originating outside the body is not a serious hazard." Or maybe a Briton would trust the BBC. The same applies to light: The skin absorbs/reflects lots of it and protects underlying tissue; while microwaves can penetrate it and reach living cells.
        – Peter A. Schneider
        yesterday














        @PeterA.Schneider Sure - emphasis on outside the body. Inside the body is a very different story. That's because our skin has been very well adapted to protect us. The point is that you can't just say "it's more penetrating -> it's more dangerous" EM Radiation isn't a gun - if it goes through you, it's harmless. Now there's a sweet spot where all the radiation is absorbed in the body, and it penetrates some distance - which is how cancer treatment works; however attempting to suggest that masters in physics is related to the media is insulting, and I'd be thankful if you could refrain.
        – UKMonkey
        yesterday




        @PeterA.Schneider Sure - emphasis on outside the body. Inside the body is a very different story. That's because our skin has been very well adapted to protect us. The point is that you can't just say "it's more penetrating -> it's more dangerous" EM Radiation isn't a gun - if it goes through you, it's harmless. Now there's a sweet spot where all the radiation is absorbed in the body, and it penetrates some distance - which is how cancer treatment works; however attempting to suggest that masters in physics is related to the media is insulting, and I'd be thankful if you could refrain.
        – UKMonkey
        yesterday












        @UKMonkey I didn't mean to insult, and I didn't know you had a Master's degree in Physics.-- "Outside the body": Well, that's the setting of the question: Obviously the radiation (light or microwave) is meant to originate outside the body, and the main reason light does less harm is that it doesn't penetrate the skin much.
        – Peter A. Schneider
        yesterday






        @UKMonkey I didn't mean to insult, and I didn't know you had a Master's degree in Physics.-- "Outside the body": Well, that's the setting of the question: Obviously the radiation (light or microwave) is meant to originate outside the body, and the main reason light does less harm is that it doesn't penetrate the skin much.
        – Peter A. Schneider
        yesterday













        10














        The danger of electromagnetic waves is a function of the photon energy, the intensity of the source and your distance from it, and the qualitative nature of the interaction of a specific frequency with organic matter.



        That latter bit is very complex. The visible spectrum, down to about infra-red, doesn't penetrate the top layer of skin, or most clothing, so for the most part its interaction is limited to heating. Strong infrared can certainly cause burns. Strong visible light can certainly cause eye damage. But very high-intensity sources of visible light are rare in daily life, and very notably, we can see them, and avoid them.



        Further down in the energy spectrum (longer wavelength) you get the "millimeter waves" of airport scanners, which can penetrate clothing but not skin, and then you're into the microwaves, uhf and vfh radio waves, and then the radio waves called shortwaves (high frequency), medium waves and longwaves (low frequency). Microwaves can penetrate into flesh, and radio frequencies can entirely traverse a human body, and these can cause very, very severe deep-tissue burns. Certain frequencies can also interfere with cardiac rhythm, which can be as fatal as it sounds.



        Your home wifi equipment produces microwaves on pretty much the same frequencies as your microwave oven, but at milliwatt power and dispersed in all directions. The oven dumps hundreds of watts into a small enclosed space. That's the difference.



        And we live in a sea of radio waves from microwave (cell phone, wifi) though uhf and vhf (two-way radios, broadcast TV and FM) and lower (broadcast radio). The key is the power. If you grab the antenna of your uncle's 500-W ham radio when he keys the mic, or climb the tower of a multi-kilowatt tv station, you'll get hurt, maybe very badly. But going about your normal business, you're probably absorbing less than a milliwatt of radio energy. And the only effect is heating, so it's little different than being in a room that's very slightly warmer.



        Now, moving upward in energy from the visible spectrum, you get ultraviolet, x-rays and then a vast spectrum of increasing energy gamma. Not only can they penetrate into flesh, they have a very specific dirty trick: they have enough photon energy to ionize molecules, and when that happens to our dna and proteins, we start to have very bad days. This is a very specific capability that begins at a certain energy threshold.



        Microwaves, as you've remarked, are in the opposite direction from UV, X-rays and gamma: lower photon energy, longer wavelength. They cannot duplicate the ionization danger of higher energies, no matter how intense their sources are.






        share|cite|improve this answer








        New contributor




        CCTO is a new contributor to this site. Take care in asking for clarification, commenting, and answering.
        Check out our Code of Conduct.























          10














          The danger of electromagnetic waves is a function of the photon energy, the intensity of the source and your distance from it, and the qualitative nature of the interaction of a specific frequency with organic matter.



          That latter bit is very complex. The visible spectrum, down to about infra-red, doesn't penetrate the top layer of skin, or most clothing, so for the most part its interaction is limited to heating. Strong infrared can certainly cause burns. Strong visible light can certainly cause eye damage. But very high-intensity sources of visible light are rare in daily life, and very notably, we can see them, and avoid them.



          Further down in the energy spectrum (longer wavelength) you get the "millimeter waves" of airport scanners, which can penetrate clothing but not skin, and then you're into the microwaves, uhf and vfh radio waves, and then the radio waves called shortwaves (high frequency), medium waves and longwaves (low frequency). Microwaves can penetrate into flesh, and radio frequencies can entirely traverse a human body, and these can cause very, very severe deep-tissue burns. Certain frequencies can also interfere with cardiac rhythm, which can be as fatal as it sounds.



          Your home wifi equipment produces microwaves on pretty much the same frequencies as your microwave oven, but at milliwatt power and dispersed in all directions. The oven dumps hundreds of watts into a small enclosed space. That's the difference.



          And we live in a sea of radio waves from microwave (cell phone, wifi) though uhf and vhf (two-way radios, broadcast TV and FM) and lower (broadcast radio). The key is the power. If you grab the antenna of your uncle's 500-W ham radio when he keys the mic, or climb the tower of a multi-kilowatt tv station, you'll get hurt, maybe very badly. But going about your normal business, you're probably absorbing less than a milliwatt of radio energy. And the only effect is heating, so it's little different than being in a room that's very slightly warmer.



          Now, moving upward in energy from the visible spectrum, you get ultraviolet, x-rays and then a vast spectrum of increasing energy gamma. Not only can they penetrate into flesh, they have a very specific dirty trick: they have enough photon energy to ionize molecules, and when that happens to our dna and proteins, we start to have very bad days. This is a very specific capability that begins at a certain energy threshold.



          Microwaves, as you've remarked, are in the opposite direction from UV, X-rays and gamma: lower photon energy, longer wavelength. They cannot duplicate the ionization danger of higher energies, no matter how intense their sources are.






          share|cite|improve this answer








          New contributor




          CCTO is a new contributor to this site. Take care in asking for clarification, commenting, and answering.
          Check out our Code of Conduct.





















            10












            10








            10






            The danger of electromagnetic waves is a function of the photon energy, the intensity of the source and your distance from it, and the qualitative nature of the interaction of a specific frequency with organic matter.



            That latter bit is very complex. The visible spectrum, down to about infra-red, doesn't penetrate the top layer of skin, or most clothing, so for the most part its interaction is limited to heating. Strong infrared can certainly cause burns. Strong visible light can certainly cause eye damage. But very high-intensity sources of visible light are rare in daily life, and very notably, we can see them, and avoid them.



            Further down in the energy spectrum (longer wavelength) you get the "millimeter waves" of airport scanners, which can penetrate clothing but not skin, and then you're into the microwaves, uhf and vfh radio waves, and then the radio waves called shortwaves (high frequency), medium waves and longwaves (low frequency). Microwaves can penetrate into flesh, and radio frequencies can entirely traverse a human body, and these can cause very, very severe deep-tissue burns. Certain frequencies can also interfere with cardiac rhythm, which can be as fatal as it sounds.



            Your home wifi equipment produces microwaves on pretty much the same frequencies as your microwave oven, but at milliwatt power and dispersed in all directions. The oven dumps hundreds of watts into a small enclosed space. That's the difference.



            And we live in a sea of radio waves from microwave (cell phone, wifi) though uhf and vhf (two-way radios, broadcast TV and FM) and lower (broadcast radio). The key is the power. If you grab the antenna of your uncle's 500-W ham radio when he keys the mic, or climb the tower of a multi-kilowatt tv station, you'll get hurt, maybe very badly. But going about your normal business, you're probably absorbing less than a milliwatt of radio energy. And the only effect is heating, so it's little different than being in a room that's very slightly warmer.



            Now, moving upward in energy from the visible spectrum, you get ultraviolet, x-rays and then a vast spectrum of increasing energy gamma. Not only can they penetrate into flesh, they have a very specific dirty trick: they have enough photon energy to ionize molecules, and when that happens to our dna and proteins, we start to have very bad days. This is a very specific capability that begins at a certain energy threshold.



            Microwaves, as you've remarked, are in the opposite direction from UV, X-rays and gamma: lower photon energy, longer wavelength. They cannot duplicate the ionization danger of higher energies, no matter how intense their sources are.






            share|cite|improve this answer








            New contributor




            CCTO is a new contributor to this site. Take care in asking for clarification, commenting, and answering.
            Check out our Code of Conduct.









            The danger of electromagnetic waves is a function of the photon energy, the intensity of the source and your distance from it, and the qualitative nature of the interaction of a specific frequency with organic matter.



            That latter bit is very complex. The visible spectrum, down to about infra-red, doesn't penetrate the top layer of skin, or most clothing, so for the most part its interaction is limited to heating. Strong infrared can certainly cause burns. Strong visible light can certainly cause eye damage. But very high-intensity sources of visible light are rare in daily life, and very notably, we can see them, and avoid them.



            Further down in the energy spectrum (longer wavelength) you get the "millimeter waves" of airport scanners, which can penetrate clothing but not skin, and then you're into the microwaves, uhf and vfh radio waves, and then the radio waves called shortwaves (high frequency), medium waves and longwaves (low frequency). Microwaves can penetrate into flesh, and radio frequencies can entirely traverse a human body, and these can cause very, very severe deep-tissue burns. Certain frequencies can also interfere with cardiac rhythm, which can be as fatal as it sounds.



            Your home wifi equipment produces microwaves on pretty much the same frequencies as your microwave oven, but at milliwatt power and dispersed in all directions. The oven dumps hundreds of watts into a small enclosed space. That's the difference.



            And we live in a sea of radio waves from microwave (cell phone, wifi) though uhf and vhf (two-way radios, broadcast TV and FM) and lower (broadcast radio). The key is the power. If you grab the antenna of your uncle's 500-W ham radio when he keys the mic, or climb the tower of a multi-kilowatt tv station, you'll get hurt, maybe very badly. But going about your normal business, you're probably absorbing less than a milliwatt of radio energy. And the only effect is heating, so it's little different than being in a room that's very slightly warmer.



            Now, moving upward in energy from the visible spectrum, you get ultraviolet, x-rays and then a vast spectrum of increasing energy gamma. Not only can they penetrate into flesh, they have a very specific dirty trick: they have enough photon energy to ionize molecules, and when that happens to our dna and proteins, we start to have very bad days. This is a very specific capability that begins at a certain energy threshold.



            Microwaves, as you've remarked, are in the opposite direction from UV, X-rays and gamma: lower photon energy, longer wavelength. They cannot duplicate the ionization danger of higher energies, no matter how intense their sources are.







            share|cite|improve this answer








            New contributor




            CCTO is a new contributor to this site. Take care in asking for clarification, commenting, and answering.
            Check out our Code of Conduct.









            share|cite|improve this answer



            share|cite|improve this answer






            New contributor




            CCTO is a new contributor to this site. Take care in asking for clarification, commenting, and answering.
            Check out our Code of Conduct.









            answered 23 hours ago









            CCTOCCTO

            2013




            2013




            New contributor




            CCTO is a new contributor to this site. Take care in asking for clarification, commenting, and answering.
            Check out our Code of Conduct.





            New contributor





            CCTO is a new contributor to this site. Take care in asking for clarification, commenting, and answering.
            Check out our Code of Conduct.






            CCTO is a new contributor to this site. Take care in asking for clarification, commenting, and answering.
            Check out our Code of Conduct.























                3














                Other answers already point out the matter of intensity. If you have a 1kW microwave that cooks your chicken and so call microwaves dangerous, you can equally cook chicken with 1kW visible light bulb. The difference is mostly how well and how deep the absorption goes, but the amount of energy is the same if the absorbed power is the same.



                What makes the difference:




                • Absorption coefficient; how WELL the light is absorbed. So, a black chicken will cook well in strong visible light, but a white chicken will require higher power, because it reflects more. Microwaves work well because they penetrate deeper before they get absorbed (due to longer wavelength), but also absorb WELL because the frequency is tuned near the resonance for water molecules.

                • Resonances; If the wavelength matches exactly to one of the transition frequencies for molecules/atoms in the target, most of the energy is selectively absorbed just by those molecules. So, if you tuned your light specifically to a transition that breaks some specific bonds, or heat just specific tissues. This can do more damage because it may change chemistry, but luckily, breaking bonds requires quite high frequencies - see next case below. With MW and IR, you will still end up just heating the sample if you find a resonance (resonances in MW, IR and visible are mostly vibrational and rotational transitions, not bond changing, except for red-colored substances reacting to visible light, which you notice when red dyes bleach quickly in intense light).

                • Ionization; If the energy of a SINGLE photon is enough to kick off an electron from a molecule/atom, then it's dangerous because it's actively affecting the chemistry (note that this is similar result than the resonant case above, but instead of having a precise frequency, it has way too much energy, with similar results). This is what is called ionising radiation (gamma/X-rays, down to UV range).


                Note that resonance just means good absorbtion, nothing mystical. Water is mostly transparent for visible light because no significant vibrations of water molecule fall in this range - most are in IR and microwave, and there is another absorption range in UV.



                Rule of thumb: microwaves, IR and visible light just heat you up. It's the heat is sufficient to raise the temperature into danger zone, it's dangerous, otherwise it's harmless. Only intensity matters (Watts per square meter), not the frequency.
                Ionizing radiation (UV/X-ray/gamma) are dangerous because of chemical damage even at low intensity.



                Microwaves are NOT ionizing radiation, so the wireless and mobile signals do absolutely nothing - the power is way too low, otherwise you'd need to charge your phone every 5 minutes.






                share|cite|improve this answer


























                  3














                  Other answers already point out the matter of intensity. If you have a 1kW microwave that cooks your chicken and so call microwaves dangerous, you can equally cook chicken with 1kW visible light bulb. The difference is mostly how well and how deep the absorption goes, but the amount of energy is the same if the absorbed power is the same.



                  What makes the difference:




                  • Absorption coefficient; how WELL the light is absorbed. So, a black chicken will cook well in strong visible light, but a white chicken will require higher power, because it reflects more. Microwaves work well because they penetrate deeper before they get absorbed (due to longer wavelength), but also absorb WELL because the frequency is tuned near the resonance for water molecules.

                  • Resonances; If the wavelength matches exactly to one of the transition frequencies for molecules/atoms in the target, most of the energy is selectively absorbed just by those molecules. So, if you tuned your light specifically to a transition that breaks some specific bonds, or heat just specific tissues. This can do more damage because it may change chemistry, but luckily, breaking bonds requires quite high frequencies - see next case below. With MW and IR, you will still end up just heating the sample if you find a resonance (resonances in MW, IR and visible are mostly vibrational and rotational transitions, not bond changing, except for red-colored substances reacting to visible light, which you notice when red dyes bleach quickly in intense light).

                  • Ionization; If the energy of a SINGLE photon is enough to kick off an electron from a molecule/atom, then it's dangerous because it's actively affecting the chemistry (note that this is similar result than the resonant case above, but instead of having a precise frequency, it has way too much energy, with similar results). This is what is called ionising radiation (gamma/X-rays, down to UV range).


                  Note that resonance just means good absorbtion, nothing mystical. Water is mostly transparent for visible light because no significant vibrations of water molecule fall in this range - most are in IR and microwave, and there is another absorption range in UV.



                  Rule of thumb: microwaves, IR and visible light just heat you up. It's the heat is sufficient to raise the temperature into danger zone, it's dangerous, otherwise it's harmless. Only intensity matters (Watts per square meter), not the frequency.
                  Ionizing radiation (UV/X-ray/gamma) are dangerous because of chemical damage even at low intensity.



                  Microwaves are NOT ionizing radiation, so the wireless and mobile signals do absolutely nothing - the power is way too low, otherwise you'd need to charge your phone every 5 minutes.






                  share|cite|improve this answer
























                    3












                    3








                    3






                    Other answers already point out the matter of intensity. If you have a 1kW microwave that cooks your chicken and so call microwaves dangerous, you can equally cook chicken with 1kW visible light bulb. The difference is mostly how well and how deep the absorption goes, but the amount of energy is the same if the absorbed power is the same.



                    What makes the difference:




                    • Absorption coefficient; how WELL the light is absorbed. So, a black chicken will cook well in strong visible light, but a white chicken will require higher power, because it reflects more. Microwaves work well because they penetrate deeper before they get absorbed (due to longer wavelength), but also absorb WELL because the frequency is tuned near the resonance for water molecules.

                    • Resonances; If the wavelength matches exactly to one of the transition frequencies for molecules/atoms in the target, most of the energy is selectively absorbed just by those molecules. So, if you tuned your light specifically to a transition that breaks some specific bonds, or heat just specific tissues. This can do more damage because it may change chemistry, but luckily, breaking bonds requires quite high frequencies - see next case below. With MW and IR, you will still end up just heating the sample if you find a resonance (resonances in MW, IR and visible are mostly vibrational and rotational transitions, not bond changing, except for red-colored substances reacting to visible light, which you notice when red dyes bleach quickly in intense light).

                    • Ionization; If the energy of a SINGLE photon is enough to kick off an electron from a molecule/atom, then it's dangerous because it's actively affecting the chemistry (note that this is similar result than the resonant case above, but instead of having a precise frequency, it has way too much energy, with similar results). This is what is called ionising radiation (gamma/X-rays, down to UV range).


                    Note that resonance just means good absorbtion, nothing mystical. Water is mostly transparent for visible light because no significant vibrations of water molecule fall in this range - most are in IR and microwave, and there is another absorption range in UV.



                    Rule of thumb: microwaves, IR and visible light just heat you up. It's the heat is sufficient to raise the temperature into danger zone, it's dangerous, otherwise it's harmless. Only intensity matters (Watts per square meter), not the frequency.
                    Ionizing radiation (UV/X-ray/gamma) are dangerous because of chemical damage even at low intensity.



                    Microwaves are NOT ionizing radiation, so the wireless and mobile signals do absolutely nothing - the power is way too low, otherwise you'd need to charge your phone every 5 minutes.






                    share|cite|improve this answer












                    Other answers already point out the matter of intensity. If you have a 1kW microwave that cooks your chicken and so call microwaves dangerous, you can equally cook chicken with 1kW visible light bulb. The difference is mostly how well and how deep the absorption goes, but the amount of energy is the same if the absorbed power is the same.



                    What makes the difference:




                    • Absorption coefficient; how WELL the light is absorbed. So, a black chicken will cook well in strong visible light, but a white chicken will require higher power, because it reflects more. Microwaves work well because they penetrate deeper before they get absorbed (due to longer wavelength), but also absorb WELL because the frequency is tuned near the resonance for water molecules.

                    • Resonances; If the wavelength matches exactly to one of the transition frequencies for molecules/atoms in the target, most of the energy is selectively absorbed just by those molecules. So, if you tuned your light specifically to a transition that breaks some specific bonds, or heat just specific tissues. This can do more damage because it may change chemistry, but luckily, breaking bonds requires quite high frequencies - see next case below. With MW and IR, you will still end up just heating the sample if you find a resonance (resonances in MW, IR and visible are mostly vibrational and rotational transitions, not bond changing, except for red-colored substances reacting to visible light, which you notice when red dyes bleach quickly in intense light).

                    • Ionization; If the energy of a SINGLE photon is enough to kick off an electron from a molecule/atom, then it's dangerous because it's actively affecting the chemistry (note that this is similar result than the resonant case above, but instead of having a precise frequency, it has way too much energy, with similar results). This is what is called ionising radiation (gamma/X-rays, down to UV range).


                    Note that resonance just means good absorbtion, nothing mystical. Water is mostly transparent for visible light because no significant vibrations of water molecule fall in this range - most are in IR and microwave, and there is another absorption range in UV.



                    Rule of thumb: microwaves, IR and visible light just heat you up. It's the heat is sufficient to raise the temperature into danger zone, it's dangerous, otherwise it's harmless. Only intensity matters (Watts per square meter), not the frequency.
                    Ionizing radiation (UV/X-ray/gamma) are dangerous because of chemical damage even at low intensity.



                    Microwaves are NOT ionizing radiation, so the wireless and mobile signals do absolutely nothing - the power is way too low, otherwise you'd need to charge your phone every 5 minutes.







                    share|cite|improve this answer












                    share|cite|improve this answer



                    share|cite|improve this answer










                    answered yesterday









                    orionorion

                    5,6781425




                    5,6781425























                        2














                        It's more accurate to say microwave ovens are dangerous. Then again, so is visible light.



                        It's a question not of photon energy, but total energy. A typical microwave outputs on the order of 1 kilowatt of electromagnetic radiation which is almost entirely absorbed by the food within.



                        By comparison, the solar power at earth's surface, at maximum, is around 1 kilowatt per square meter. If it's cloudy, not at the equator, or not noon, it will be less. Most foods have a surface area of much less than a square meter, so the total electromagnetic radiation power received by something sitting in the sun is much less than a microwave oven.



                        For a fair comparison, what do you think would happen if a magnifying glass with an area of one square meter, on a very sunny day, were used to focus light on to something the size of what you'd put into a microwave oven?



                        There are a few more subtle differences. For example dangerous powers of visible light are so bright you'll surely close your eyes. Furthermore, visible light penetrates less deeply, so you're likely to feel the heat and move away before it does any more than superficial damage to your skin, like a sunburn. On the other hand, microwave radiation is invisible and penetrates more deeply, so you may suffer irreversible injury before even noticing the hazard. The cornea is especially prone to microwave injury since there's no protective reflex to protect it, it has little thermal mass and thus heats quickly, and there's little blood flow to cool it.






                        share|cite|improve this answer























                        • Of course, solar ovens are a thing - and indeed, to get a decent oven, a square meter of mirrors concentrating the sunlight to the center (with the food item) is more than enough.
                          – Luaan
                          4 hours ago
















                        2














                        It's more accurate to say microwave ovens are dangerous. Then again, so is visible light.



                        It's a question not of photon energy, but total energy. A typical microwave outputs on the order of 1 kilowatt of electromagnetic radiation which is almost entirely absorbed by the food within.



                        By comparison, the solar power at earth's surface, at maximum, is around 1 kilowatt per square meter. If it's cloudy, not at the equator, or not noon, it will be less. Most foods have a surface area of much less than a square meter, so the total electromagnetic radiation power received by something sitting in the sun is much less than a microwave oven.



                        For a fair comparison, what do you think would happen if a magnifying glass with an area of one square meter, on a very sunny day, were used to focus light on to something the size of what you'd put into a microwave oven?



                        There are a few more subtle differences. For example dangerous powers of visible light are so bright you'll surely close your eyes. Furthermore, visible light penetrates less deeply, so you're likely to feel the heat and move away before it does any more than superficial damage to your skin, like a sunburn. On the other hand, microwave radiation is invisible and penetrates more deeply, so you may suffer irreversible injury before even noticing the hazard. The cornea is especially prone to microwave injury since there's no protective reflex to protect it, it has little thermal mass and thus heats quickly, and there's little blood flow to cool it.






                        share|cite|improve this answer























                        • Of course, solar ovens are a thing - and indeed, to get a decent oven, a square meter of mirrors concentrating the sunlight to the center (with the food item) is more than enough.
                          – Luaan
                          4 hours ago














                        2












                        2








                        2






                        It's more accurate to say microwave ovens are dangerous. Then again, so is visible light.



                        It's a question not of photon energy, but total energy. A typical microwave outputs on the order of 1 kilowatt of electromagnetic radiation which is almost entirely absorbed by the food within.



                        By comparison, the solar power at earth's surface, at maximum, is around 1 kilowatt per square meter. If it's cloudy, not at the equator, or not noon, it will be less. Most foods have a surface area of much less than a square meter, so the total electromagnetic radiation power received by something sitting in the sun is much less than a microwave oven.



                        For a fair comparison, what do you think would happen if a magnifying glass with an area of one square meter, on a very sunny day, were used to focus light on to something the size of what you'd put into a microwave oven?



                        There are a few more subtle differences. For example dangerous powers of visible light are so bright you'll surely close your eyes. Furthermore, visible light penetrates less deeply, so you're likely to feel the heat and move away before it does any more than superficial damage to your skin, like a sunburn. On the other hand, microwave radiation is invisible and penetrates more deeply, so you may suffer irreversible injury before even noticing the hazard. The cornea is especially prone to microwave injury since there's no protective reflex to protect it, it has little thermal mass and thus heats quickly, and there's little blood flow to cool it.






                        share|cite|improve this answer














                        It's more accurate to say microwave ovens are dangerous. Then again, so is visible light.



                        It's a question not of photon energy, but total energy. A typical microwave outputs on the order of 1 kilowatt of electromagnetic radiation which is almost entirely absorbed by the food within.



                        By comparison, the solar power at earth's surface, at maximum, is around 1 kilowatt per square meter. If it's cloudy, not at the equator, or not noon, it will be less. Most foods have a surface area of much less than a square meter, so the total electromagnetic radiation power received by something sitting in the sun is much less than a microwave oven.



                        For a fair comparison, what do you think would happen if a magnifying glass with an area of one square meter, on a very sunny day, were used to focus light on to something the size of what you'd put into a microwave oven?



                        There are a few more subtle differences. For example dangerous powers of visible light are so bright you'll surely close your eyes. Furthermore, visible light penetrates less deeply, so you're likely to feel the heat and move away before it does any more than superficial damage to your skin, like a sunburn. On the other hand, microwave radiation is invisible and penetrates more deeply, so you may suffer irreversible injury before even noticing the hazard. The cornea is especially prone to microwave injury since there's no protective reflex to protect it, it has little thermal mass and thus heats quickly, and there's little blood flow to cool it.







                        share|cite|improve this answer














                        share|cite|improve this answer



                        share|cite|improve this answer








                        edited 58 mins ago

























                        answered 19 hours ago









                        Phil FrostPhil Frost

                        3,36421726




                        3,36421726












                        • Of course, solar ovens are a thing - and indeed, to get a decent oven, a square meter of mirrors concentrating the sunlight to the center (with the food item) is more than enough.
                          – Luaan
                          4 hours ago


















                        • Of course, solar ovens are a thing - and indeed, to get a decent oven, a square meter of mirrors concentrating the sunlight to the center (with the food item) is more than enough.
                          – Luaan
                          4 hours ago
















                        Of course, solar ovens are a thing - and indeed, to get a decent oven, a square meter of mirrors concentrating the sunlight to the center (with the food item) is more than enough.
                        – Luaan
                        4 hours ago




                        Of course, solar ovens are a thing - and indeed, to get a decent oven, a square meter of mirrors concentrating the sunlight to the center (with the food item) is more than enough.
                        – Luaan
                        4 hours ago











                        0














                        The trick with microwaves is that they use resonance frequencies of water. Some microwaves can be tuned to meat, vegetables or fish, since the resonance frequency can change slightly in composition. Visible light does not resonate with anything in our bodies.



                        Our generally photo-sensitive skin mostly reacts to UV-Ranges, even in a not yet ionizing spectrum. (The wavelength also determines how deep the light can enter the skin!)



                        With enough energy, visible light could be dangerous, but the energy needed is far greater without any resonance effects. (think of a child swing)






                        share|cite|improve this answer








                        New contributor




                        Anonymous is a new contributor to this site. Take care in asking for clarification, commenting, and answering.
                        Check out our Code of Conduct.


















                        • Welcome to Physics SE! Hope you'll enjoy it here. Great first answer.
                          – Andrea
                          yesterday






                        • 9




                          Microwaves don't use resonance; it's a common misconception. See explanation e.g. here or anywhere on the Internet.
                          – user27542
                          yesterday










                        • In what is fish water different from meat water or vegetable water? Not to mention that you can heat up butter just fine in a microwave.
                          – Dmitry Grigoryev
                          yesterday












                        • @user27542 Interesting fact, didn't know. Why does visible light not do any dielectric heating? Maybe it's not resonance in the strict sense but you must at least be in the frequency ballpark?
                          – Peter A. Schneider
                          yesterday








                        • 1




                          @DmitryGrigoryev -- butter has a lot of water in it. When you put butter in a hot pan, the bubbles are the water boiling off.
                          – Pete Becker
                          yesterday


















                        0














                        The trick with microwaves is that they use resonance frequencies of water. Some microwaves can be tuned to meat, vegetables or fish, since the resonance frequency can change slightly in composition. Visible light does not resonate with anything in our bodies.



                        Our generally photo-sensitive skin mostly reacts to UV-Ranges, even in a not yet ionizing spectrum. (The wavelength also determines how deep the light can enter the skin!)



                        With enough energy, visible light could be dangerous, but the energy needed is far greater without any resonance effects. (think of a child swing)






                        share|cite|improve this answer








                        New contributor




                        Anonymous is a new contributor to this site. Take care in asking for clarification, commenting, and answering.
                        Check out our Code of Conduct.


















                        • Welcome to Physics SE! Hope you'll enjoy it here. Great first answer.
                          – Andrea
                          yesterday






                        • 9




                          Microwaves don't use resonance; it's a common misconception. See explanation e.g. here or anywhere on the Internet.
                          – user27542
                          yesterday










                        • In what is fish water different from meat water or vegetable water? Not to mention that you can heat up butter just fine in a microwave.
                          – Dmitry Grigoryev
                          yesterday












                        • @user27542 Interesting fact, didn't know. Why does visible light not do any dielectric heating? Maybe it's not resonance in the strict sense but you must at least be in the frequency ballpark?
                          – Peter A. Schneider
                          yesterday








                        • 1




                          @DmitryGrigoryev -- butter has a lot of water in it. When you put butter in a hot pan, the bubbles are the water boiling off.
                          – Pete Becker
                          yesterday
















                        0












                        0








                        0






                        The trick with microwaves is that they use resonance frequencies of water. Some microwaves can be tuned to meat, vegetables or fish, since the resonance frequency can change slightly in composition. Visible light does not resonate with anything in our bodies.



                        Our generally photo-sensitive skin mostly reacts to UV-Ranges, even in a not yet ionizing spectrum. (The wavelength also determines how deep the light can enter the skin!)



                        With enough energy, visible light could be dangerous, but the energy needed is far greater without any resonance effects. (think of a child swing)






                        share|cite|improve this answer








                        New contributor




                        Anonymous is a new contributor to this site. Take care in asking for clarification, commenting, and answering.
                        Check out our Code of Conduct.









                        The trick with microwaves is that they use resonance frequencies of water. Some microwaves can be tuned to meat, vegetables or fish, since the resonance frequency can change slightly in composition. Visible light does not resonate with anything in our bodies.



                        Our generally photo-sensitive skin mostly reacts to UV-Ranges, even in a not yet ionizing spectrum. (The wavelength also determines how deep the light can enter the skin!)



                        With enough energy, visible light could be dangerous, but the energy needed is far greater without any resonance effects. (think of a child swing)







                        share|cite|improve this answer








                        New contributor




                        Anonymous is a new contributor to this site. Take care in asking for clarification, commenting, and answering.
                        Check out our Code of Conduct.









                        share|cite|improve this answer



                        share|cite|improve this answer






                        New contributor




                        Anonymous is a new contributor to this site. Take care in asking for clarification, commenting, and answering.
                        Check out our Code of Conduct.









                        answered yesterday









                        AnonymousAnonymous

                        411




                        411




                        New contributor




                        Anonymous is a new contributor to this site. Take care in asking for clarification, commenting, and answering.
                        Check out our Code of Conduct.





                        New contributor





                        Anonymous is a new contributor to this site. Take care in asking for clarification, commenting, and answering.
                        Check out our Code of Conduct.






                        Anonymous is a new contributor to this site. Take care in asking for clarification, commenting, and answering.
                        Check out our Code of Conduct.












                        • Welcome to Physics SE! Hope you'll enjoy it here. Great first answer.
                          – Andrea
                          yesterday






                        • 9




                          Microwaves don't use resonance; it's a common misconception. See explanation e.g. here or anywhere on the Internet.
                          – user27542
                          yesterday










                        • In what is fish water different from meat water or vegetable water? Not to mention that you can heat up butter just fine in a microwave.
                          – Dmitry Grigoryev
                          yesterday












                        • @user27542 Interesting fact, didn't know. Why does visible light not do any dielectric heating? Maybe it's not resonance in the strict sense but you must at least be in the frequency ballpark?
                          – Peter A. Schneider
                          yesterday








                        • 1




                          @DmitryGrigoryev -- butter has a lot of water in it. When you put butter in a hot pan, the bubbles are the water boiling off.
                          – Pete Becker
                          yesterday




















                        • Welcome to Physics SE! Hope you'll enjoy it here. Great first answer.
                          – Andrea
                          yesterday






                        • 9




                          Microwaves don't use resonance; it's a common misconception. See explanation e.g. here or anywhere on the Internet.
                          – user27542
                          yesterday










                        • In what is fish water different from meat water or vegetable water? Not to mention that you can heat up butter just fine in a microwave.
                          – Dmitry Grigoryev
                          yesterday












                        • @user27542 Interesting fact, didn't know. Why does visible light not do any dielectric heating? Maybe it's not resonance in the strict sense but you must at least be in the frequency ballpark?
                          – Peter A. Schneider
                          yesterday








                        • 1




                          @DmitryGrigoryev -- butter has a lot of water in it. When you put butter in a hot pan, the bubbles are the water boiling off.
                          – Pete Becker
                          yesterday


















                        Welcome to Physics SE! Hope you'll enjoy it here. Great first answer.
                        – Andrea
                        yesterday




                        Welcome to Physics SE! Hope you'll enjoy it here. Great first answer.
                        – Andrea
                        yesterday




                        9




                        9




                        Microwaves don't use resonance; it's a common misconception. See explanation e.g. here or anywhere on the Internet.
                        – user27542
                        yesterday




                        Microwaves don't use resonance; it's a common misconception. See explanation e.g. here or anywhere on the Internet.
                        – user27542
                        yesterday












                        In what is fish water different from meat water or vegetable water? Not to mention that you can heat up butter just fine in a microwave.
                        – Dmitry Grigoryev
                        yesterday






                        In what is fish water different from meat water or vegetable water? Not to mention that you can heat up butter just fine in a microwave.
                        – Dmitry Grigoryev
                        yesterday














                        @user27542 Interesting fact, didn't know. Why does visible light not do any dielectric heating? Maybe it's not resonance in the strict sense but you must at least be in the frequency ballpark?
                        – Peter A. Schneider
                        yesterday






                        @user27542 Interesting fact, didn't know. Why does visible light not do any dielectric heating? Maybe it's not resonance in the strict sense but you must at least be in the frequency ballpark?
                        – Peter A. Schneider
                        yesterday






                        1




                        1




                        @DmitryGrigoryev -- butter has a lot of water in it. When you put butter in a hot pan, the bubbles are the water boiling off.
                        – Pete Becker
                        yesterday






                        @DmitryGrigoryev -- butter has a lot of water in it. When you put butter in a hot pan, the bubbles are the water boiling off.
                        – Pete Becker
                        yesterday













                        0














                        When we say that microwaves are less energetic, we are talking about the energy in a single photon. The number of photons is also important.



                        A single microwave photon is utterly harmless. Its only effect is heat, and it takes a serious amount of heat to damage us. But enough heat, in any form, will kill.



                        Visible light has enough energy that single photons can cause chemical reactions, but only in sensitive compounds. That is what happens in our eyes. The chemicals in our eyes are carefully constructed to be extra sensitive to light and that is what makes light visible.



                        Ultraviolet light is worse. Here the photons carry enough energy to cause unwanted chemical reactions in most organic compounds. Sun burn and skin cancer results.



                        Gamma ray photons from radioactivity are even worse, but they are fortunately rare.






                        share|cite|improve this answer


























                          0














                          When we say that microwaves are less energetic, we are talking about the energy in a single photon. The number of photons is also important.



                          A single microwave photon is utterly harmless. Its only effect is heat, and it takes a serious amount of heat to damage us. But enough heat, in any form, will kill.



                          Visible light has enough energy that single photons can cause chemical reactions, but only in sensitive compounds. That is what happens in our eyes. The chemicals in our eyes are carefully constructed to be extra sensitive to light and that is what makes light visible.



                          Ultraviolet light is worse. Here the photons carry enough energy to cause unwanted chemical reactions in most organic compounds. Sun burn and skin cancer results.



                          Gamma ray photons from radioactivity are even worse, but they are fortunately rare.






                          share|cite|improve this answer
























                            0












                            0








                            0






                            When we say that microwaves are less energetic, we are talking about the energy in a single photon. The number of photons is also important.



                            A single microwave photon is utterly harmless. Its only effect is heat, and it takes a serious amount of heat to damage us. But enough heat, in any form, will kill.



                            Visible light has enough energy that single photons can cause chemical reactions, but only in sensitive compounds. That is what happens in our eyes. The chemicals in our eyes are carefully constructed to be extra sensitive to light and that is what makes light visible.



                            Ultraviolet light is worse. Here the photons carry enough energy to cause unwanted chemical reactions in most organic compounds. Sun burn and skin cancer results.



                            Gamma ray photons from radioactivity are even worse, but they are fortunately rare.






                            share|cite|improve this answer












                            When we say that microwaves are less energetic, we are talking about the energy in a single photon. The number of photons is also important.



                            A single microwave photon is utterly harmless. Its only effect is heat, and it takes a serious amount of heat to damage us. But enough heat, in any form, will kill.



                            Visible light has enough energy that single photons can cause chemical reactions, but only in sensitive compounds. That is what happens in our eyes. The chemicals in our eyes are carefully constructed to be extra sensitive to light and that is what makes light visible.



                            Ultraviolet light is worse. Here the photons carry enough energy to cause unwanted chemical reactions in most organic compounds. Sun burn and skin cancer results.



                            Gamma ray photons from radioactivity are even worse, but they are fortunately rare.







                            share|cite|improve this answer












                            share|cite|improve this answer



                            share|cite|improve this answer










                            answered yesterday









                            Stig HemmerStig Hemmer

                            21924




                            21924























                                0














                                There is nothing inherently more or less dangerous about microwaves. Yes, the type of damage via microwave vs. visible light vs. xray are different but weather or not the light cause damage has the same factor for all the spectrum of light - intensity.



                                Remember, modern computer-using humans constantly volunteer to be bather in microwaves. Wifi uses exactly the same frequency as microwave ovens. The difference between wifi and microwave ovens is the wattage - the amount of power used to generate the light - the intensity.



                                You can cook with visible light if you pump enough power into it - or somehow concentrate it. This is how sun ovens work and how you can burn paper with a magnifying glass. You can also cook with xrays if it is intense enough.



                                Side note: Most people don't realise this but high intensity infrared light can blind you as surely as staring at the sun. Just because it is invisible to your eyes does not mean the photons does not hit your retina. High intensity infrared spotlights are sold as part of security systems for infrared sensitive cameras (night vision).






                                share|cite|improve this answer





















                                • "High intensity" is relative, of course; most spotlights used for "night vision cameras" are still just a few watts, which is rather small compared to sunlight. There is a danger mainly because you don't realize you're too close to the light (and there's no eye response to the increasing amount of light hitting your eyes), and if it's dark, your pupils are maximally dilated. You don't see microwaves, but above about 20 W, the heat is quite noticeable, and a 200 W light is unmistakably warm (those are usually used for heating, though, not IR camera illumination).
                                  – Luaan
                                  4 hours ago
















                                0














                                There is nothing inherently more or less dangerous about microwaves. Yes, the type of damage via microwave vs. visible light vs. xray are different but weather or not the light cause damage has the same factor for all the spectrum of light - intensity.



                                Remember, modern computer-using humans constantly volunteer to be bather in microwaves. Wifi uses exactly the same frequency as microwave ovens. The difference between wifi and microwave ovens is the wattage - the amount of power used to generate the light - the intensity.



                                You can cook with visible light if you pump enough power into it - or somehow concentrate it. This is how sun ovens work and how you can burn paper with a magnifying glass. You can also cook with xrays if it is intense enough.



                                Side note: Most people don't realise this but high intensity infrared light can blind you as surely as staring at the sun. Just because it is invisible to your eyes does not mean the photons does not hit your retina. High intensity infrared spotlights are sold as part of security systems for infrared sensitive cameras (night vision).






                                share|cite|improve this answer





















                                • "High intensity" is relative, of course; most spotlights used for "night vision cameras" are still just a few watts, which is rather small compared to sunlight. There is a danger mainly because you don't realize you're too close to the light (and there's no eye response to the increasing amount of light hitting your eyes), and if it's dark, your pupils are maximally dilated. You don't see microwaves, but above about 20 W, the heat is quite noticeable, and a 200 W light is unmistakably warm (those are usually used for heating, though, not IR camera illumination).
                                  – Luaan
                                  4 hours ago














                                0












                                0








                                0






                                There is nothing inherently more or less dangerous about microwaves. Yes, the type of damage via microwave vs. visible light vs. xray are different but weather or not the light cause damage has the same factor for all the spectrum of light - intensity.



                                Remember, modern computer-using humans constantly volunteer to be bather in microwaves. Wifi uses exactly the same frequency as microwave ovens. The difference between wifi and microwave ovens is the wattage - the amount of power used to generate the light - the intensity.



                                You can cook with visible light if you pump enough power into it - or somehow concentrate it. This is how sun ovens work and how you can burn paper with a magnifying glass. You can also cook with xrays if it is intense enough.



                                Side note: Most people don't realise this but high intensity infrared light can blind you as surely as staring at the sun. Just because it is invisible to your eyes does not mean the photons does not hit your retina. High intensity infrared spotlights are sold as part of security systems for infrared sensitive cameras (night vision).






                                share|cite|improve this answer












                                There is nothing inherently more or less dangerous about microwaves. Yes, the type of damage via microwave vs. visible light vs. xray are different but weather or not the light cause damage has the same factor for all the spectrum of light - intensity.



                                Remember, modern computer-using humans constantly volunteer to be bather in microwaves. Wifi uses exactly the same frequency as microwave ovens. The difference between wifi and microwave ovens is the wattage - the amount of power used to generate the light - the intensity.



                                You can cook with visible light if you pump enough power into it - or somehow concentrate it. This is how sun ovens work and how you can burn paper with a magnifying glass. You can also cook with xrays if it is intense enough.



                                Side note: Most people don't realise this but high intensity infrared light can blind you as surely as staring at the sun. Just because it is invisible to your eyes does not mean the photons does not hit your retina. High intensity infrared spotlights are sold as part of security systems for infrared sensitive cameras (night vision).







                                share|cite|improve this answer












                                share|cite|improve this answer



                                share|cite|improve this answer










                                answered 13 hours ago









                                slebetmanslebetman

                                21026




                                21026












                                • "High intensity" is relative, of course; most spotlights used for "night vision cameras" are still just a few watts, which is rather small compared to sunlight. There is a danger mainly because you don't realize you're too close to the light (and there's no eye response to the increasing amount of light hitting your eyes), and if it's dark, your pupils are maximally dilated. You don't see microwaves, but above about 20 W, the heat is quite noticeable, and a 200 W light is unmistakably warm (those are usually used for heating, though, not IR camera illumination).
                                  – Luaan
                                  4 hours ago


















                                • "High intensity" is relative, of course; most spotlights used for "night vision cameras" are still just a few watts, which is rather small compared to sunlight. There is a danger mainly because you don't realize you're too close to the light (and there's no eye response to the increasing amount of light hitting your eyes), and if it's dark, your pupils are maximally dilated. You don't see microwaves, but above about 20 W, the heat is quite noticeable, and a 200 W light is unmistakably warm (those are usually used for heating, though, not IR camera illumination).
                                  – Luaan
                                  4 hours ago
















                                "High intensity" is relative, of course; most spotlights used for "night vision cameras" are still just a few watts, which is rather small compared to sunlight. There is a danger mainly because you don't realize you're too close to the light (and there's no eye response to the increasing amount of light hitting your eyes), and if it's dark, your pupils are maximally dilated. You don't see microwaves, but above about 20 W, the heat is quite noticeable, and a 200 W light is unmistakably warm (those are usually used for heating, though, not IR camera illumination).
                                – Luaan
                                4 hours ago




                                "High intensity" is relative, of course; most spotlights used for "night vision cameras" are still just a few watts, which is rather small compared to sunlight. There is a danger mainly because you don't realize you're too close to the light (and there's no eye response to the increasing amount of light hitting your eyes), and if it's dark, your pupils are maximally dilated. You don't see microwaves, but above about 20 W, the heat is quite noticeable, and a 200 W light is unmistakably warm (those are usually used for heating, though, not IR camera illumination).
                                – Luaan
                                4 hours ago


















                                draft saved

                                draft discarded




















































                                Thanks for contributing an answer to Physics Stack Exchange!


                                • Please be sure to answer the question. Provide details and share your research!

                                But avoid



                                • Asking for help, clarification, or responding to other answers.

                                • Making statements based on opinion; back them up with references or personal experience.


                                Use MathJax to format equations. MathJax reference.


                                To learn more, see our tips on writing great answers.





                                Some of your past answers have not been well-received, and you're in danger of being blocked from answering.


                                Please pay close attention to the following guidance:


                                • Please be sure to answer the question. Provide details and share your research!

                                But avoid



                                • Asking for help, clarification, or responding to other answers.

                                • Making statements based on opinion; back them up with references or personal experience.


                                To learn more, see our tips on writing great answers.




                                draft saved


                                draft discarded














                                StackExchange.ready(
                                function () {
                                StackExchange.openid.initPostLogin('.new-post-login', 'https%3a%2f%2fphysics.stackexchange.com%2fquestions%2f452592%2fif-visible-light-has-more-energy-than-microwaves-why-isnt-visible-light-danger%23new-answer', 'question_page');
                                }
                                );

                                Post as a guest















                                Required, but never shown





















































                                Required, but never shown














                                Required, but never shown












                                Required, but never shown







                                Required, but never shown

































                                Required, but never shown














                                Required, but never shown












                                Required, but never shown







                                Required, but never shown







                                Popular posts from this blog

                                An IMO inspired problem

                                Management

                                Has there ever been an instance of an active nuclear power plant within or near a war zone?